You are on page 1of 34

20.

EVA LUATION OF COEXISTING DISEASE

Jacob T. Arnett and Chien-Hsiang Chow

1. A 7-month-old female is referred for a transthoracic B. Psychiatric disorder characterized by impairment in


echo due to a murmur that her pediatrician auscul- development of culturally normal relationships
tated on physical exam. A 0.4 cm ASD is visualized at C. Organic disorder characterized by
the center of the intra-atrial septum and is believed to developmental delay
be an ostium secundum defect. The echo report shows a D. Organic disorder characterized by seizures of varying
pulmonary-to-systemic blood flow (Qp:Qs) of 1.2. The frequency
most appropriate treatment at this time is:
4. An 11-year-old male with a history of dyskinetic cere-
A. Open surgical closure as right heart failure is likely bral palsy is seen with his parents prior to placement of
in the near future a baclofen intrathecal pump. The mother relays that no
B. Medical therapy with indomethacin medications were taken for 2 days prior to surgery, as he
C. Observation refused to take them. A peripheral IV is placed in the
D. Percutaneous closure to avoid Eisenmenger’s induction area and dexmedetomodine is started as a pre-
syndrome in the future medication for anxiolysis. Five minutes later the nurs-
ing staff requests your immediate presence. The child
2. A 1-year-old-male with a history of Tetralogy of Fallot is unresponsive and appears to be having a tonic-clonic
presents for a sedated transthoracic echocardiogram. seizure. The most likely cause of the seizure is:
His cardiac surgeon is in the process of surgical plan-
ning and needs a repeat echocardiogram. During an A. Side effect from dexmetetomidine
IM induction the child becomes hypoxic and cyanotic. B. Electrolyte abnormality
You quickly flex his hips and knees and begin a phenyl- C. Subacute meningitis
ephrine infusion, in addition to a fluid bolus. His blood D. Medication withdrawal
pressure is stable but his oxygen saturations are slow to
increase. The next step in pharmacologic management 5. Succinylcholine use is likely considered acceptable in
includes which of the following? which of the following patients?

A. Small boluses of esmolol A. A 7-year-old male having an open-reduction, internal


B. Deepening the anesthetic by increasing the inhaled fixation of a supracondylar humeral fracture whose
anesthetic concentration mother has a history of malignant hyperthermia
C. Starting a milrinone infusion to reduce pulmonary B. A 13-year-old female with a history of a 70%
artery pressures BSA deep burn sustained 5 months ago having a
D. Small boluses of epinephrine to increase systemic contracture release
vascular resistance and reduce pulmonary pressures C. A 9-year-old male with a history of spastic cerebral
palsy having an Achilles tendon lengthening
3. Which of the following best describes autism? procedure
D. An 11-year-old male with notable
A. Psychiatric disorder characterized by culturally pseudohypertrophy of the calves scheduled for a
inappropriate behaviors muscle biopsy under general anesthesia


6. A  9-year-old male with a history of cystic fibro- the time of surgery and was resuscitated in the pediatric
sis is evaluated prior to laparoscopic appendectomy. intensive care unit (PICU). The PICU team reports his
Preoperative labs show an elevated PT and INR but electrolytes have normalized, and he appears euvolemic.
normal PTT and platelet values. What is the most likely In reviewing the child’s labs you notice his hemoglobin
cause of the elevated PT/INR? is 10.1 g/dL. This hemoglobin level:

A. Intestinal malabsorption A. Likely reflects an iron-deficient anemia from


B. Warfarin use malabsorption/chronic vomiting
C. Perioperative enoxeparin use B. Likely reflects a gastrointestinal bleed with acute
D. Hemophilia A blood loss anemia, which will require blood
transfusions in the operating room
7. A  2-year-old male with a history of Trisomy 21 is C. Likely reflects hemodilution from the IV fluid
scheduled for an ophthalmic exam under anesthesia. resuscitation
With regard to planning airway management, which of D. Likely represents a normal lab finding for a
the following statements regarding the airway manage- 9-week-old male
ment is true?
10. A  9-month-old male is seen in preop prior to a
A. Microglossia may make for a challenging direct circumcision, which is being completed for cosmetic
laryngoscopy. reasons. His parents report a normal perinatal his-
B. Hypertonia may be present, causing upper airway tory with no major problems since the time of birth.
obstruction. During a review of systems, his father reports that
C. Macrognathia is a common finding. he has been able to sit alone and recently showed the
D. Airway manipulation may be limited due to ability to stand up. He babbles but has yet to speak
potential for cervical spine instability. any words. He smiles and plays “peek-a-boo” with
his mother but does not seem to play with his twin
8. A 3-year-old child with recurrent tonsillitis is evalu- brother. The above developmental status of this child
ated prior to a tonsillectomy. The parents report that the is likely:
child’s pediatrician noted a heart murmur at a recent
visit, but they have no other details. The mother relates A. Normal and appropriate for his age
that the child is doing very well and she did not have any B. Globally abnormal and indicates
concerns prior to the pediatrician’s visit. You observe developmental delay
the child smiling and appearing comfortable in her C. Motor development is normal but social
mother’s arms. When auscultating the heart, you appre- development is delayed
ciate a II/VI systolic murmur at the left-lower sternal D. Social development is normal but motor
border. The child moves in her mother’s arms and the development is delayed
murmur becomes very faint and difficult to appreciate.
Which of the following is most likely true? 11. With regard to child and parental preoperative
anxiety and emotional stress in the preoperative period,
A. The child should be referred for an which of the following is true?
electrocardiogram (ECG) and a transthoracic echo
prior to anesthesia. A. Children younger than 7 years experience less
B. The murmur may not be significant during activities anxiety than older children because they do not
of daily living (ADLs) but will likely be an issue understand the danger of anesthesia/surgery.
during an inhalational induction when systemic B. Midazolam given for preoperative anxiety gives
vascular resistance decreases. satisfactory anxiolysis and sedation in approximately
C. The murmur is most likely not clinically significant one third of children.
and she can proceed to surgery without further C. Discussion of specific anesthetic risks will
diagnostic testing. significantly increase parental anxiety in the
D. The murmur likely represents significant underlying preoperative period.
cardiac pathology; she should be sent to the D. Child temperament is generally not indicative of
emergency room for admission and a complete preoperative anxiety and coping strategies.
workup.
12. A 4-year-old female has a broken arm and is sched-
9. A 9-week-old male is scheduled for a pyloromyotomy uled for an open reduction internal fixation of the
for pyloric stenosis. He was admitted 20 hours prior to humerus. During the preoperative evaluation the

  r  $ - * / * $ " -  4 $ * & / $ &  0 ' ु " / & 4 5 ) & 4 * "


mother requests that she be present during induction C. Type 3 hypersensitivity reaction
of anesthesia. What could be done to adequately pre- D. Type 4 hypersensitivity reaction
pare the mother for her presence during the induction
of anesthesia? 16. Neonates and infants with gastroesophageal reflux
disease (GERD) can present with which of the follow-
A. Nothing, it is not routinely recommended ing symptoms?
B. Explaining that the most upsetting areas for parents
are when the child goes limp and when the parent A. Prolonged PT/INR
has to leave the child B. Failure to thrive
C. Telling the mother that she should be quiet during C. Renal insufficiency
induction and not to distract her daughter as she D. Rhinorrhea
goes to sleep
D. Explaining to the mother that studies have shown 17. An 8-month-old infant presents to the hospital
unequivocally that pediatric patient anxiety is lowest for repair of an inguinal hernia. Other than moderate
with a parent present during induction GERD, the patient is healthy but was born at 29 weeks
gestation. Which of the following would be appropriate
13. An 8-year-old male with an anterior mediastinal premedication and/or intervention in this child prior to
mass comes to the preop clinic for preoperative evalu- induction of anesthesia?
ation prior to a mediastinoscopy. On evaluation the
patient is able to tolerate lying supine, but you notice A. Proceeding with surgery 4 hours after breast milk
that he has distended neck veins, facial swelling, and B. Famotidine
has been complaining of difficulty breathing. Which of C. Pantoprazole
the following test/s should be ordered in the evaluation D. Magnesium
of this mass?
18. Which of the following is most consistent with
A. Exercise electrocardiogram asthma and management of asthma?
B. Gastric motility study
C. Positron emission tomography (PET) scan A. Wheezing is produced by a combination of upper
D. Pulmonary function tests airway inflammation and bronchospasm.
B. Appropriate therapy includes bronchodilators,
14. A  12-year-old male with an anterior mediastinal anti-inflammatories, mast cell stabilizers, and
mass is scheduled for a mediastinoscopy. On evalua- leukotriene inhibitors.
tion the patient is able to tolerate laying supine and the C. Immunoglobulin D is associated with the disease
patient’s preoperative echocardiogram shows no signs process.
of vascular compression. Which of the following is the D. Manifestations of the disease include accessory
most appropriate technique to induce anesthesia in this muscle use, coughing, and prolonged inspiration.
patient?
19. A 1-year-old is to undergo a laparoscopic Nissen fun-
A. Rapid sequence induction with cricoid pressure doplication in the morning. Three hours prior to surgery
B. Intravenous induction with propofol the patient was given formula by her parents. What is the
C. Inhalational induction appropriate fasting interval for this particular situation?
D. Intravenous opioid-based induction for
hemodynamic stability A. 2 hours
B. 4 hours
15. An 8-month-old with a history of spina bifida C. 6 hours
occulta is scheduled for a repair of a umbilical hernia. D. 8 hours
During the initial history the patient’s family states
that the child has had rashes after dental procedures. 20. Which of the following decreases the risk of respi-
During the procedure the child suddenly develops ratory complications in the setting of upper respiratory
hives, becomes hypotensive, and difficult to ventilate. infection?
What type of hypersensitivity reaction is responsible
for the patient’s symptoms? A. Delaying surgery 3 weeks
B. Using an LMA in place of an endotracheal tube
A. Type 1 hypersensitivity reaction C. Preoperative beta agonists (albuterol)
B. Type 2 hypersensitivity reaction D. Cavity, head, or neck surgery

    & 7" - 6"5 * 0 /  0 ' ु $ 0 & 9 * 4 5 * / (  % * 4 & " 4 &  r  


21. Select the statement most true regarding anesthetic B. Children under the age of 3 years receive a great
risk: deal of benefit from preparation programs aimed at
enhancing coping skills.
A. Risk for both major and minor adverse event in C. Prior hospitalization routinely aids in calming
pediatric patients exceeds that for adult patients. children undergoing surgery.
B. Nausea is the most common adverse event in all D. Parental anxiety is strongly associated with child
pediatric patients. anxiety.
C. Children between the ages of 1 and 12 months are at
greatest risk for major adverse event among pediatric 26. Which severe disease state is commonly associated
patients. with seizure disorder and is strongly associated with
D. Children between the ages of 1 and 12 months impaired oromotor function and recurrent aspiration
undergo cardiac surgery more frequently than other pneumonia?
pediatric patients.
A. Idiopathic scoliosis
22. Please select which of the following qualities is true B. Cerebral palsy
in the context of pediatric patient consent? C. Cystic fibrosis
D. Trisomy 21
A. Children younger than 7 years of age are typically
capable of assenting due to their decision-making 27. An 11-year-old Caucasian male with cystic fibro-
ability. sis presents with abdominal pain and is scheduled for
B. Between the ages of 7 and 14 years, children reliably a laparoscopic appendectomy. Which of the following
develop decision-making skills. treatment plans optimizes his postoperative outcomes?
C. Children above the age of 14 years may reliably hold
adequate decision-making skills. A. Cessation of preoperative bronchodilators
D. Children above the age of 14 years have developed B. Prophylactic broad spectrum antibiotics
the cognitive and emotional skills required to fully C. Incentive spirometry and postural drainage
participate in the process of consent. D. Low-calorie diet, continuation of pancreatic enzyme
replacement, and fat-soluble vitamin supplements
23. A 14-year-old male presents to a preoperative clinic
2 weeks prior to a planned knee arthroscopy. The 28. A  2-month-old infant who was born at 30 weeks
patient had myringotomy with drainage tubes placed at gestation with complete bladder extrophy presents for
2 years of age without incident. The patient has no med- exploratory laparotomy and intestinal stricture resec-
ical issues aside from his knee. What would be the most tion. This is her fourteenth general anesthetic. Once a
appropriate assessment tool?
new Foley catheter is placed in the operating room, her
electrocardiogram (EKG) demonstrates sinus tachycar-
A. Complete blood count and basic metabolic panel
dia, a noninvasive blood pressure cannot be obtained,
B. Complete blood count and electrocardiogram and the ventilator alarms because her peak airway pres-
C. Complete blood count sures acutely increase >40  cm H2O. Which of the fol-
D. No lab assessment lowing is most important in treating this patient?
24. Select the answer which is most consistent with
parental presence on induction of anesthesia (PPI). A. Administer H1 and H2 blockers
B. Administer epinephrine
A. PPI is absolutely contraindicated above the patient C. Administer corticosteroid
age of 18 years. D. Send blood specimen for mast cell tryptase level
B. PPI is a benign process for the parent.
C. A possible benefit of PPI is reduction in need for 29. A  13-year-old otherwise healthy boy presents for
preoperative sedation. resection of an anterior mediastinal mass. He and his
D. A possible benefit of PPI is reduction of parent anxiety. parents note that for the last 9 months, he has not been
able to participate during recess at school because of
25. Choose the correct statement regarding psychoso- progressive tiredness, he has a constant dry cough, and
cial preparation of pediatric patients preoperatively. he uses two pillows to sleep comfortably at night. As
you examine him, you notice plethora of his neck and
A. Children aged 5–10 years receive little benefit from face. Which of the following preoperative studies best
preparation programs aimed at enhancing coping detects evidence of dynamic airway compression in this
skills. patient?

   r  $ - * / * $ " -  4 $ * & / $ &  0 ' ु " / & 4 5 ) & 4 * "


A. Chest X-ray weight gain with no sweating or cyanotic episodes with
B. CT scan feeds, and she walks without difficulty in the preop-
C. Cardiac echocardiogram erative holding area. As you examine the patient, you
D. Pulmonary function tests notice Down facies, macrocephaly, macroglossia, and
you hear a holosystolic murmur on auscultation. What
30. A talkative 7-year-old boy with Williams syndrome should you do next?
presents for tonsillectomy and adenoidectomy for sleep
apnea and recurrent tonsillitis. Which of the follow- A. Obtain a chest X-ray.
ing findings is most alarming during your preoperative B. Obtain an EKG and cardiac echocardiogram.
assessment? C. Obtain a cardiac stress test.
D. Proceed with surgery.
A. Moderate aortic stenosis
B. Rhinorrhea and dry cough 34. A  13-year-old severely autistic adolescent presents
C. Micrognathia with prominent incisors for dental rehabilitation. Dentists have been unsuccess-
D. 4 + tonsils ful in examining his teeth in the office because he is a
“biter.” He has poor dental hygiene because he will not
31. A 3-year-old boy with Tetralogy of Fallot is sched- allow anyone to brush his teeth or place anything near
uled for urgent left groin exploration with possible his face. Which of the following options is most reliable
orchiopexy versus orchiectomy for left testicular tor- in inducing general anesthesia in this severely autistic
sion. He has had groin pain for 4 hours and arrives from patient?
the emergency department with three failed intravenous
attempts. As he begins to cry in the preoperative hold- A. Intravenous induction with propofol and fentanyl
ing area, you notice his lips turning blue. Bearing in B. Inhalational induction with sevoflurane
mind hemodynamic goals for a patient with Tetralogy C. Oral midazolam, then inhalational induction with
of Fallot, what should you do next? sevoflurane
D. Intramuscular ketamine, then inhalational
A. Administer oral midazolam and provide oxygen by induction with sevoflurane
facemask.
B. Administer intramuscular ketamine and provide 35. A 14-year-old girl with asthma and idiopathic sco-
oxygen by facemask. liosis with a Cobb angle of 60 degrees presents for
C. Place a peripheral intravenous to increase his T2-pelvis posterior spinal fusion with instrumentation.
intravascular volume. She appears thin, tearful, and nervous in the preopera-
D. Find a junior partner to do the case for you. tive area. Her parents inform you that they are Jehovah’s
Witnesses and do not want any transfusion of blood or
32. If this same 3-year-old boy with Tetralogy of Fallot blood products to their daughter. She is in the middle
and testicular torsion is now in the operating room of menses now. What is your most important periop-
and is crying, what is the best way to safely attenuate a erative concern for this patient and her upcoming major
hypercyanotic episode in this patient with Tetralogy of surgery?
Fallot and no intravenous access?
A. Discussion of anxiolytic strategy
A. Use EMLA, then place an intravenous to administer B. Discussion of blood conservation strategies
an anxiolytic, crystalloid, and phenylephrine. C. Discussion of neuromonitoring and possible
B. Proceed with mask induction with cricoid pressure, wake-up test
high inspired concentration of sevoflurane, and D. Discussion of susceptibility to malignant
100% FiO2 . hyperthermia
C. Proceed with mask induction with cricoid pressure,
high inspired concentration of sevoflurane, and 80% 36. A neonate with hypertelorism, midface hypoplasia,
N2O/20% O2 . seizures, and tetany presents for omphalocele repair.
D. Administer intramuscular ketamine and provide Given the nature of this major surgery, which will likely
supplemental oxygen. involve large fluid shifts and potential transfusion
of blood products, which of the following is the most
33. A  chubby 2-year-old with Down syndrome has important part of your preoperative evaluation?
severe obstructive sleep apnea with an apnea-hypopnea
index of 42 and presents for tonsillectomy and adenoid- A. Focused airway exam
ectomy. She feeds well, her parents report appropriate B. Cardiovascular workup

    & 7" - 6"5 * 0 /  0 ' ु $ 0 & 9 * 4 5 * / (  % * 4 & " 4 &  r  


C. Evaluation of electrolyte abnormalities 40. An ex-31-week, now 4-year-old boy presents for
D. Ensuring appropriate urine output circumcision for phimosis. He has received no vacci-
nations and has been febrile for the past 36 hours. As
37. A full-term 3 kg neonate male with a large, prena- you examine him in the preoperative holding area, you
tally diagnosed myelomeningocele presents for myelo- observe rhinorrhea, hear diffuse rales in his lung fields
meningocele repair and placement of a VP shunt for bilaterally, and see the patient cough up yellow sputum.
hydrocephalus. What is your primary reasoning for How do you proceed?
recovering this patient in an ICU-level setting?
A. Proceed with GA, use an LMA to avoid intubation,
A. Fluid resuscitation supplement with a dorsal penile block.
B. Postoperative respiratory monitoring B. Administer albuterol in preop holding, then proceed
C. Proper patient positioning with GA.
D. Maintaining euthermia C. Administer steroids and azithromycin, then proceed
with GA.
38. An ex-28-week, now 6-week-old twin B has a unilat- D. Postpone this elective procedure for at least 4 weeks.
eral inguinal hernia. He is now 4 kg. You administered a
single-shot spinal with 0.4 cc 0.5% bupivacaine for your 41. A  16-year-old, African American male with sickle
anesthetic, in your efforts to avoid post-operative apnea cell disease presents for laparoscopic cholecystectomy.
in this former preterm infant. After the hernia sac has He is maintained on chronic folic acid and hydroxyurea
been dissected, the infant starts moving his legs. Since therapy. Which of the following would be most delete-
he is under the surgical drapes, you cannot repeat the rious to him in the perioperative period?
spinal. Which of the following techniques safely pre-
vents postoperative apnea in this baby? A. Liberal use of analgesics
B. Transfusion of phenotype-matched,
A. Ask the surgeon to supplement with local to the leukocyte-reduced blood to target hemoglobin
surgical field. 10 g/dL
B. Induce general anesthesia but use fentanyl instead of C. Hyperventilation to EtCO2 < 25 mm Hg
morphine. D. Intraoperative use of inhaled nitric oxide
C. Induce general anesthesia but administer no narcotic.
D. Supplement with ketamine to keep the baby 42. A  5-year-old girl with Beckwith-Wiedemann syn-
breathing spontaneously. drome has hemihypertrophy and presents for left distal
femur epiphysiodesis. Her parents report that she had
39. A full-term, now 6-week-old firstborn male presents a perinatal omphalocele repair with no complications,
with 4  days of nonbilious projectile vomiting, and an and that all of her follow-up screens for somatic tumors
ultrasound confirms hypertrophic pyloric stenosis. He have been negative. You plan a combined general anes-
is scheduled for laparoscopic pyloromyotomy for hyper- thetic and femoral nerve block catheter for postopera-
trophic pyloric stenosis. Which of the following sets of tive analgesia. Which of the following studies would
laboratory values best indicates that this infant is ready most strongly influence your perioperative care for this
for surgery? patient?
Na+ K+ Cl– HCO3–
A. Fingerstick glucose
A. 130 3.1 89 34 B. Liver function tests
B. 142 3.8 105 25 C. Cardiac echocardiogram
C. 150 3.2 102 32 D. Caffeine-contracture testing for malignant
D. 142 3.6 92 28 hyperthermia susceptibility

  r  $ - * / * $ " -  4 $ * & / $ &  0 ' ु " / & 4 5 ) & 4 * "


$) " 15 &3ु" /48 &3 4 eds. Gregory’s Pediatric Anesthesia. 5th ed. Hoboken, NJ: Blackwell;
2011:642–3.
Franco, Samantha A.; Hines, Roberta L. “Congenital Heart Disease”
in Hines RL, Marschall KE, eds. Stoelting’s Anesthesia and
1. A N S W E R : C Co-Existing Disease. 6th ed. Philadelphia, PA:  Saunders Elsevier;
2012:49–51.
Atrial septal defects (ASDs) are common congenital cardiac
lesions and can vary in severity and timing of presentation.
The vast majority (75%) of ASDs are ostium secundum 2 . A N S W E R : A
defects that occur at the center of the interatrial septum in
the region formerly occupied by the foramen ovale. Septum Tetralogy of Fallot (TOF) is the most common congenital
primum ASDs are due to endocardial cushion defects, cyanotic lesion, accounting for 10% of congenital heart dis-
which leave a large opening in the center of interatrial sep- ease cases. The four components of the lesion are (1) a VSD,
tum. There are also sinus venosum defects, which are rarer. (2) an “overriding” aorta that covers ventricles, (3) RVOT
In the absence of other major cardiac anomalies, ASDs obstruction, and (4) right ventricular hypertrophy (RVH).
will initially be acyanotic lesions as blood generally shunts This child is experiencing a hypercyanotic “Tet” spell, indi-
left to right. Small ASDs, such as in this question, are gener- cating that the volume of blood shunting from right to
ally watched and not intervened upon. They typically create left has increased. After increasing preload (fluid bolus,
a systolic ejection murmur at the left upper sternal bor- flexing the hips) and increasing afterload (flexing the hips
der, which may be mistaken for a flow murmur, particularly and using phenylephrine), the next step is to reduce the
in a well-appearing child. Larger lesions (e.g., approaching obstruction to RV outflow. Though it may seem logical to
2  cm in diameter) can lead to significant shunting caus- reduce PA pressures with milrinone or epinephrine, the
ing increased flow through the pulmonary circulation. distal pulmonary pressures are generally low and not a sig-
An echo that reveals a Qp:Qs of > 1.5 signifies significant nificant factor in the degree of shunting. The obstruction
shunting and may warrant closure, either open or by percu- to RV outflow is at the level of the RVOT. This may be
taneous intervention. The long-term consequences include subvalvular, valvular, supravalvular, and occasionally at
right-heart failure, pulmonary hypertension, and if the pulmonary arterial branches. Esmolol will slow heart
untreated, reversal of the shunt if right-heart pressure rate to allow for adequate preload and will also relax the
exceeds the pressure in the left heart. infundibular (subvalvular) region of the RVOT, allowing
Anesthetic considerations for patients with ASDs are more blood to leave the RV into the RVOT. For children
based around management of the SVR:PVR ratio. If the with recurrent Tet spells, propanolol is often indicated for
PVR drops considerably, there will be increased shunting outpatient use.
and worsened right-heart signs/symptoms. Conversely, An anesthetic for a child with TOF must be well
if the PVR is allowed to rise due to hypoxia, hypercarbia, planned and tailored to the patient. Premedication is often
acidosis, or hypothermia, this may lead to transient devel- recommended as crying and struggling can worsen breath-
opment of a cyanotic lesion. It is imperative to remember ing and cause desaturations. Midazolam has been shown to
than even if only left-to-right shunting is present on echo, be effective. The induction of anesthesia on patients with
there is still a risk of paradoxical emboli. Many patients do TOF can cause significant hemodynamic changes, which
well for years, and the ASD may actually be undetected, in can quickly lead to cyanosis. Ketamine is often used as
which case they may not present until adulthood. an induction agent. The increase in SVR with ketamine
will reduce the right-to-left shunt. An inhaled induction
is acceptable, but one must remember that the reduction in
K E Y FAC T S SVR will cause an increase in the shunt. Historically, halo-
r In the absence of other major cardiac anomalies, ASDs thane has been the preferred inhalational agent for induc-
will initially be acyanotic lesions as blood generally tion as it maintains a relatively stable SVR but causes direct
shunts left to right. myocardial depression, helping to relax the infundibulum.
r Larger lesions with a Qp:Qs of > 1.5 and significant Although still in use in some centers, the popularity of hal-
shunting may lead to right-heart failure, pulmonary othane has decreased due to the risk of fulminant hepatic
hypertension, and reversal of shunt flow if untreated. necrosis. Positive-pressure ventilation, hypercarbia,
r Anesthetic considerations for patients with ASDs are and hypoxia will raise pulmonary pressures and increase
rooted in management of the SVR:PVR ratio. the right-to-left shunt, so effective airway management is
imperative when balancing SVR and PVR. Care should
also be taken to avoid high airway pressures and allow ade-
3 & ' & 3 & /$ & 4 quate time for expiration to avoid increased intrathoracic
pressures. As with all cardiac lesions involving shunts, care
Odegard, Kirsten C., DiNardo, James A., Laussen, Peter C. “Anesthesia must be taken to avoid air in the IV tubing due to risk of
for Congenital Heart Disease” in Gregory GA, Andropoulos DB, paradoxical emboli.

    & 7" - 6"5 * 0 /  0 ' ु $ 0 & 9 * 4 5 * / (  % * 4 & " 4 &  r 


K E Y FAC T S Emergence from general anesthesia is a particularly dif-
ficult time for autistic patients. Changes in sedation level,
r TOF is the most common congenital cyanotic lesion
coupled with abrupt changes in location, present a challenge
and consists of a VSD, an “overriding” aorta, RVOT
to all children, but these are particularly challenging for
obstruction, and RVH.
autistic children. Coupled with surgical pain, temperature
r A child experiencing a “tet” spell has increased right-to-
changes, and hunger, emergence and transition through the
left shunting, and is treated by increasing preload and
postanesthetic care unit can be a time of particular diffi-
afterload as well as reducing the RVOT obstruction.
culty with autistic patients.
This can be accomplished by giving beta blockers.
r Positive-pressure ventilation, hypercarbia, and hypoxia
will raise pulmonary pressures and increase the right-to- K E Y FAC T S
left shunt, so effective airway management is imperative
r Autism is a poorly understood psychiatric disorder
when balancing SVR and PVR.
that manifests in multiple fashions, most centrally as
an inability to develop interpersonal relationships as
considered normal in a given culture.
3 & ' & 3 & /$ & 4
r Behaviors vary among autistic patients and could
Odegard, Kirsten C., DiNardo, James A., Laussen, Peter C. “Anesthesia be largely considered as either hyperreactive or
for Congenital Heart Disease” in Gregory GA, Andropoulos DB, hyporeactive. For hyperreactive patients, it is important
eds. Gregory’s Pediatric Anesthesia 5th ed. Hoboken, NJ: Blackwell; to explore the triggers for agitation (lighting, touch,
2011:633–5. sounds, etc.) and calming techniques from the parents.
Franco, Samantha A.; Hines, Roberta L. “Congenital Heart Disease” in
Hines RL, Marschall KE, eds. Stoelting’s Anesthesia and Co-Existing r Coupled with surgical pain, temperature changes,
Disease. 6th ed. Philadelphia, PA: Saunders Elsevier; 2012:57–9. and hunger, emergence and transition through the
Levine MF, Hartley EJ, Macpherson BA, Burrows FA, Lerman J. Oral postanesthetic care unit can be a time of particular
midazolam premedication for children with congenital cyanotic difficulty with autistic patients.
heart disease undergoing cardiac surgery: a comparative study. Can J
Anaesth. 1993;40(10):934–8.

3 & ' & 3 & /$ &


3 . A N S W E R : B
Diu, Michelle W.; Mancuso, Thomas J. “Pediatric Disease” in Stoelting
Autism is a poorly understood psychiatric disorder that RK, Dierdorf SF, eds. Anesthesia and Co-Existing Disease. 4th ed.
manifests in multiple fashions, most centrally as an Philadelphia, PA: Churchill Livingstone; 2002:639.
inability to develop interpersonal relationships as
considered normal in a given culture. Associated findings
4 . A N S W E R : D
may include developmental delay, seizures, and culturally
inappropriate behaviors but are not central to the diagno- Cerebral palsy (CP) is a nonprogressive neurologic disorder
sis. Behaviors vary among autistic patients and could be that is multifactorial in origin but is generally related to a
largely considered as either hyperreactive or hyporeac- variety of prenatal, perinatal, and postnatal insults. The
tive. Among hyperreactive patients, it is paramount that disease process is not well understood, and contributing
the anesthesiologist inquire about triggers for patients in factors can range from prematurity to infection to intrace-
order to provide a safe environment. Likewise, it is impor- rebral hemorrhage. Perinatal asphyxia was once thought to
tant to distinguish hyporeactive behaviors because they be the most common etiology but is now believed to cause
mimic common perioperative issues such as hypoglyce- less than 10% of CP. The key feature of CP is a disorder
mia, anesthetic overdose, and hypothermia. Currently no of movement and posture. There may be varying degrees of
curative treatment exists for autism. Life span is normal mental retardation and developmental delay, depending on
with continuation of childhood symptoms into adult- the etiology of the cerebral palsy. Multiple classification sys-
hood being relatively common. tems have been developed for CP; a commonly used system
Substantial consideration must be given to the details is based on the type of neurologic impairment (i.e., spastic,
of the operative experience when the patient is autistic in dyskinetic, atheoid, mixed). Spastic CP is the most com-
order to deliver the safest experience possible. Details such mon, accounting for 70% of cases.
as lighting, stimulation, calming techniques, and agita- Children with CP frequently have seizure disorders
tion triggers are most easily explored with the parent. In with an incidence of at least 30%. They are often on antiepi-
addition, child life specialists are tremendously helpful in leptic drugs, in addition to antispasmodic medications, and
this arena, freeing the anesthesiologist to devote mental withdrawal can trigger seizures as in this scenario. In the
resources to medical concerns such as preoperative sedation perioperative period it is important to review all medica-
and pain control. tions and administer preoperatively, if necessary.

  r  $ - * / * $ " -  4 $ * & / $ &  0 ' ु " / & 4 5 ) & 4 * "


Dexmedetomidine is an alpha-2 agonist often used as K E Y FAC T S
an adjunct for general anesthesia. It has potent sedative r Conditions that cause upper or lower motor neuron
effects and provides analgesia, which can be opioid sparing.
defects, burns, and prolonged immobilization can lead
There have been no case reports of seizures/convulsions in
to upregulation of ACh receptors, causing hyperkalemia
humans.
after succinylcholine use.
r Despite the neuromuscular disease, succinylcholine use
K E Y FAC T S does not seem to cause hyperkalemia in CP patients.
r If there is concern for malignant hyperthermia,
r CP is a nonprogressive neurologic disorder that is
succinylcholine should be avoided.
multifactorial in origin but is generally related to a
variety of prenatal, perinatal, and postnatal insults. It is a
disorder of movement and posture, with varying degrees 3 & ' & 3 & /$ & 4
of mental retardation and developmental delay.
r Spastic CP is the most common, accounting for 70% Parness, Jerome; Lerman, Jerrold; Stough, Robert C. “Malignant
of cases, and other types are based on the neurologic Hyperthermia” in Cote C, Lerman J, Todres ID, eds. A Practice
of Anesthesia for Infants and Children. 4th ed. Philadelphia,
impairment (i.e., spastic, dyskinetic, atheoid, mixed). PA: Saunders Elsevier; 2009:847–66.
r Children with CP frequently have seizure disorders with Lerman J. Perioperative management of the paediatric patient with coex-
an incidence of at least 30%. isting neuromuscular disease. Br J Anaesth. 2011;107(S1):i79–89.
Martyn J, Richtsfeld M. Succinylcholine-induced hyperkalemia in
acquired pathologic states. Anesthesiology. 2006;104:158–69.
3 & ' & 3 & /$ & 4
6 . A N S W E R : A
Loepke, Andreas W.; Davidson, Andrew J. “Surgery, Anesthesia, and the
Immature Brain” in Cote CJ, Lerman J, Todres ID, eds. A Practice Cystic fibrosis (CF) is an autosomal recessive disorder that
of Anesthesia for Infants and Children. 4th ed. Philadelphia, PA: affects 1 in 2000 Caucasian births in the United States and
Saunders Elsevier; 2009:492–4. is the most common fatal inherited disease in white indi-
Diu, Michelle W.; Mancuso, Thomas J. “Pediatric Disease” in
Hines RL, Marschall KE, eds. Stoelting’s Anesthesia and viduals. The genetic basis for the disease is a mutation in the
Co-Existing Disease. 6th ed. Philadelphia, PA: Saunders Elsevier; CFTR gene on chromosome 7. This gene codes for a chlo-
2012:604–5. ride transporter located on epithelial cells in several organs,
Lerman J. Perioperative management of the paediatric patient with coex- including the lungs, pancreas, liver, and gastrointestinal
isting neuromuscular disease. Br J Anaesth. 2011;107(S1):i79–89.
Miller RD, Eriksson LI, Fleisher LA, Wiener-Kronish JP, Young tract. Decreased chloride transport results in decreased
WL, eds. Miller’s Anesthesia. 7th ed. Philadelphia, PA:  Chuchill sodium and water transport into the luminal surface of the
Livingstone; 2010:752–4, 2248. organs. This yields thick, dehydrated secretions that cause
luminal obstruction with subsequent inflammation and
5 . A N S W E R : C scarring. The lungs are generally affected at a young age
with decreased clearance of the thick secretions being the
Gastroesophageal reflux disease (GERD) is quite com- impetus for recurrent bacterial infections, with later find-
mon in patients with cerebral palsy (CP) and clinicians ings of bronchiectasis and emphysema. The lungs account
often opt for an RSI prior to intubation. Due to CP being for nearly 90% of the morbidity and mortality of CF.
a neuromuscular disease, anesthesia trainees often assume The GI tract is affected with pancreatic exocrine insuf-
depolarizing NMBs are contraindicated, but this is not ficiency, poor GI motility, and altered enterohepatic cir-
true. There have been no case reports of hyperkalemia after culation. More than 90% of CF patients have pancreatic
succinylcholine administration despite many years of use. dysfunction, and most are prone to recurrent episodes of
This is due to only minimal upregulation of ACh recep- pancreatitis. Due to malabsorption and liver dysfunction,
tors around the neuromuscular junction despite having CF patients may have vitamin K deficiency and reduced
spastic paresis. levels of clotting factors II, VII, IX, and X.  The deficien-
Answers C (burns) and D (myopathy that is classic for cies would prolong the PT but generally not the PTT and
Duchenne muscular dystrophy) represent conditions that would not affect platelet levels. In the absence of clinically
may lead to hyperkalemia following succinylcholine use due significant bleeding, there are no screening labs that must
to upregulation of ACh receptors. be obtained prior to surgery, but one should keep vitamin
Malignant hyperthermia (MH) is a condition involv- K deficiency in mind when evaluating a patient with CF.
ing abnormal intracellular handling of calcium, often
related to the ryanodine receptor at the sarcoplasmic reticu- K E Y FAC T S
lum. Succinylcholine is one of the known triggering agents
and should be avoided in patients with a known or sus- r CF is an autosomal recessive disorder that affects 1
pected family history of MH, as in Answer B. in 2000 Caucasian births in the United States and

    & 7" - 6"5 * 0 /  0 ' ु $ 0 & 9 * 4 5 * / (  % * 4 & " 4 &  r  


is the most common fatal inherited disease in white K E Y FAC T S
individuals. r Down syndome (Trisomy 21) is a common occurrence
r Decreased chloride transport results in decreased
and is likely to be encountered by a pediatric
sodium and water transport into the luminal surface of
anesthesiologist.
the organs, yielding thick, dehydrated secretions that r Airway abnormalities include brachycephaly,
cause obstruction with subsequent inflammation and
macroglossia, micrognathia, hypotonia, and subglottic
scarring.
tracheal stenosis.
r Due to malabsorption and liver dysfunction, CF r Cervical spine laxity may be present, and a focused
patients may have vitamin K deficiency and reduced
history and physical as well as review of the medical
levels of clotting factors II, VII, IX and X, causing a
record are important to avoid subluxation.
prolonged PT.

3 & ' & 3 & /$ & 4


3 & ' & 3 & /$ & 4
American Academy of Pediatrics. Health supervision for children with
Al-Ruzzeh, Sharif; Kurup, Viji. “Respiratory Diseases” in Hines RL, Down syndrome. Pediatrics. 2011;128(2):393–406.
Marschall KE, eds. Stoelting’s Anesthesia and Co-Existing Disease. Weaver, R.  Grey Jr., Tobin, Joseph R. “Opthalmology” in Cote C,
6th ed. Philadelphia, PA: Saunders Elsevier; 2012:196. Lerman J, Todres ID, eds. A Practice of Anesthesia for Infants and
Huffmyer JL Littlewood KE, Nemergut EC. Perioperative man- Children 4th ed. Philadelphia, PA: Saunders Elsevier; 2009:688–9.
agement of the adult with cystic fibrosis. Anesth Analg. Diu, Michelle W.; Mancuso, Thomas J. “Pediatric Disease” in Hines
2009;109(6):1949–61. RL, Marschall KE, eds. Stoelting’s Anesthesia and Co-Existing
Disease 6th ed. Philadelphia, PA: Saunders Elsevier; 2012:634–634.

7. A N S W E R : D
8 . A N S W E R : C
Down syndrome (Trisomy 21) is relatively common occur-
ring in 1:700–800 live births, and affected children may Murmurs are often appreciated on children during pre-
present for a variety of procedures. In addition to a com- operative examinations, and the pediatric anesthesiolo-
plete history and physical, one should concentrate on the gist will frequently be tasked with guiding what, if any,
airway examination. Brachycephaly, macroglossia, micro- diagnostic testing must be completed prior to undergoing
gnathia, hypotonia, and subglottic tracheal stenosis are surgery. In this scenario, the murmur likely represents a
common findings, which may make positioning, direct so-called innocent murmur. The term “innocent” implies
laryngoscopy, and successful intubation difficult. The nar- that there are no cardiac abnormalities present; the mur-
rowed subglottic diameter makes postintubation croup mur is caused by flow through normal anatomic structures.
more common, so consideration should be given to using a Because these are flow dependent, they will change with
smaller endotracheal tube. position or maneuvers that alter venous return and car-
In addition to airway abnormalities, there may be con- diac output. In contrast, a fixed murmur would imply a
siderable laxity in the cervical spine, with both occiput-C1 structural abnormality. Innocent murmurs are exclusively
and C1-C2 instability having been described. Parents systolic, not graded more than III/VI on auscultation.
should be questioned about the child’s history of cervi- A common example is Still’s murmur, which is low pitched
cal spine instability, complaints of neck pain, inability and typically appreciated at the left lower sternal bor-
to move the neck, gait abnormalities, sensorimotor defi- der without radiation across the precordium. A  Valsalva
cits, and bowel/bladder incontinence. The medical record maneuver will greatly reduce the murmur or make it dis-
should be reviewed for prior cervical spine films, which appear entirely. No clicks or other abnormal heart sounds
typically include lateral flexion-extension plain films. can be appreciated with Still’s murmur. The rest of the
Symptomatic children are screened via radiographs physical exam, including the pulmonary exam, peripheral
between 3 and 5 years of age in accordance with recom- pulses, and signs of normal perfusion (e.g., capillary refill
mendations from the American Academy of Pediatrics, and skin color) should be normal.
while the guidelines do not recommend routine radio- The evaluation of the murmur should begin with a good
graphs of asymptomatic children. The anesthesiologist history from the child’s parents. They should be questioned
should be careful to minimize neck hyperextension about the birth history, perinatal events, issues with feed-
during mask ventilation or direct laryngoscopy, even in ing, episodes of cyanosis, ability to perform physical tasks
the absence of a history that suggests cervical instability. similar to children of their age, and any complaints from
Positioning during the case should also be a point of con- the child related to chest pain, syncope/presyncope, or dys-
cern, particularly during head/neck procedures where the pnea. A family history of significant heart disease should be
surgeon may ask for a position that could compromise the elicited. Children with a completely normal history are less
cervical spine. likely to have significant underlying cardiac pathology.

  r  $ - * / * $ " -  4 $ * & / $ &  0 ' ु " / & 4 5 ) & 4 * "


In this case, the fact that the child’s movement changed the abnormal lab values are often ignored without being
the intensity of the murmur and her general state of followed by repeat studies or interventions to correct the
well-being make an innocent murmur the most likely etiol- abnormalities. In addition to adding to increasing health
ogy, and further workup is generally not required prior to care costs, the unwarranted testing exposes the child to
anesthesia. undue stress and needle sticks. Despite these findings, sig-
nificant numbers of physicians continue to order screening
labs prior to surgery.
K E Y FAC T S
r Innocent murmurs are exclusively systolic, not graded
K E Y FAC T S
more than III/VI on auscultation. A common example
is Still’s murmur, which is low pitched and typically r Term neonates have elevated Hb levels in the
appreciated at the left lower sternal border without 15–20 g/dL range.
radiation across the precordium. r Due to reduced EPO levels and a reduction in HbF
r The rest of the physical exam, including the pulmonary synthesis, Hb levels drop to 10–12 g/dL with the nadir
exam, peripheral pulses, and signs of normal perfusion in the 8- to 12-week range.
(e.g., capillary refill and skin color) should be normal. r A further workup for this physiologic anemia is not
generally indicated.
r Routine laboratory testing for minor procedures is
3 & ' & 3 & /$ & 4 not warranted unless there are specific concerns to be
addressed.
Biancaniello T. Innocent murmurs. Circulation. 2005;111:e20–e22.
Slesnick, Timothy C.; Gertler, Ralph; Miller-Hance, Wanda C.
“Essentials of Cardiology” in Cote CJ, Lerman J, Todres ID, eds. A
Practice of Anesthesia for Infants and Children. 4th ed. Philadelphia, 3 & ' & 3 & /$ & 4
PA: Saunders Elsevier; 2009:307–8.
Litman RS. Basics of Pediatric Anesthesia. Tablet ed. Philadelphia, Ghazal, Elizabeth A.; Mason, Linda J.; Coté, Charles J. “Preoperative
PA: Ronald S. Litman; 2013:118–9. Evaluation, Premedication, and Induction of Anesthesia” in Cote
CJ, Lerman J, Todres ID, eds. A Practice of Anesthesia for Infants and
Children. 4th ed. Philadelphia, PA: Saunders Elsevier; 2009:59–62.
9. A N S W E R : D Motoyama, Etsuro K.; Finder, Jonathan D. “Respiratory Physiology
in Infants and Children” in Davis PJ, Cladis FP, Motoyama EK,
Though all of the choices are possible, hemoglobin (Hb) of eds. Smith’s Anesthesia for Infants and Children. 8th ed. St. Louis,
MO: Mosby Elsevier; 2011:395–417.
10 g/dL represents a normal finding for an infant at 9 weeks Miller, Bruce E.; Hendrickson, Jeanne E. “Coagulation, Bleeding, and
of age. A term neonate typically has a hemoglobin level of Blood Transfusion” in Gregory GA, Andropoulos DB, eds. Gregory’s
17–18 g/dL. This is caused by very high erythropoietin Pediatric Anesthesia. 5th ed. Hoboken, NJ: Blackwell; 2011:233–4.
(EPO) levels in fetal circulation due to the low oxygen ten- O’Connor ME, Drasner K. Preoperative laboratory testing of children
undergoing elective surgery. Anesth Analg. 1990;70:176–80.
sion in the uterine environment and the leftward position Patel RI, DeWitt L, Hannallah RS. Preoperative laboratory testing in
of the fetal hemoglobin-oxygen dissociation curve. The children undergoing elective surgery:  analysis of current practice.
elevated EPO levels cause Hb levels considerably higher J Clin Anesth. 1997;9(7):569–75.
than in older children or adults. As oxygen levels rise in the
neonate, there is a sharp decline in EPO and hematopoiesis
10 . A N S W E R : A
is reduced. In addition, fetal hemoglobin (HbF) production
is reduced as adult hemoglobin (HbA) production begins to The scenario represents normal development for a healthy
rise. The nadir of the resultant anemia is generally seen at child. Inquiring about the development of a pediatric patient
8–12 weeks of age with Hb levels in the 10–12 g/dL range. is extremely important in assessing the child’s overall health
This is referred to as a physiologic anemia, indicating that status. One very commonly used screening method to assess
normal physiology accounts for the laboratory values with- development makes use of milestones as defined by the Denver
out underlying pathology. Developmental Screening Test (DDST). The milestones
Of note, there are no guidelines that have established are described in broad categories:  gross motor (e.g., sitting,
“safe” Hb levels for pediatric surgery. For routine cases not standing), fine motor (pincer grasp, playing with blocks), lan-
expected to cause significant bleeding, checking hemo- guage (e.g., talking), and personal-social (e.g., smiling, separa-
globin in healthy children prior to surgery is generally tion anxiety, playing with others). The American Academy of
not indicated. Several authors have studied preoperative Pediatrics recommends a developmental assessment using the
Hb/Hct values in large numbers of children and found milestones at 9 months, 18 months, and 30 months, but many
very low rates of anemia. Preoperative coagulation studies practitioners assess aspects of developmental at all well-child
rarely delay surgery or change management; the utility of visits. When used as a screening method, the sensitivity and
these tests is often questioned. Evidence also indicates that specificity of the milestones are >70%.

    & 7" - 6"5 * 0 /  0 ' ु $ 0 & 9 * 4 5 * / (  % * 4 & " 4 &  r  


While the anesthesiologist may not frequently com- preparation 5–7 days prior to surgery experienced low lev-
plete a full developmental screening test, understanding els of anxiety, whereas those who underwent the program
the expected norms based on a child’s age will help in plan- 24 hours preoperatively experienced the highest levels of
ning an anesthetic tailored to the child. This will make for a anxiety. Distraction techniques such as music, television,
smoother induction, avoid postoperative behavioral issues, and video games are also valuable methods for reducing
and potentially change the pharmacologic approach to the anxiety.
anesthetic. Behavioral and developmental delays are incred- Premedication is commonly used, with midazolam
ibly common, present in 15%–18% of pediatric patients. and clonidine being the best studied agents. Midazolam is
It is important to remember that while missing a specific effective due to its sedating and anxiolytic effects, as well as
milestone may not affect an anesthetic plan, it may be a clue the anterograde amnesia that results from administration.
that a larger issue is present. A recent study reported satisfactory results in 75% of chil-
Table 8–1 in Nelson’s Essentials of Pediatrics, sixth edi- dren who were given oral midazolam preoperatively. Other
tion (copyright 2011), is a good overview of the milestones studies have shown that there may be a higher incidence of
and provides the salient information without excessive postoperative behavioral issues, when midazolam has been
detail. administered, so one must be careful to use it only when
indicated.

3 & ' & 3 & /$ & 4


K E Y FAC T S
Niezgoda, Julie. “Behavioral Development” in Davis PJ, Cladis FP, r Up to 50% of all children undergoing anesthesia will
Motoyama EK, eds. Smith’s Anesthesia for Infants and Children. 8th
ed. St. Louis, MO: Mosby Elsevier; 2011:10–21.
have high levels of anxiety regarding the process.
Levine, David A. “Growth and Development” in Kliegman RM, r Children younger than 7 years generally experience
Behrman RE, Marcdante K, eds. Nelson’s Essentials of Pediatrics. more anxiety than older children, as the older children
6th ed. Philadelphia, PA: Saunders Elsevier; 2011:19–21. tend to develop coping mechanisms to deal with the
anxiety.
11. A N S W E R : D r Midazolam is effective due to its sedating and anxiolytic
effects, as well as the anterograde amnesia that
Preoperative anxiety and the need for proper psychosocial results from administration. A recent study reported
preparation represents one of the most important aspects of satisfactory results in 75% of children who were given
pediatric anesthesia. Up to 50% of all children undergoing oral midazolam preoperatively.
anesthesia will have high levels of anxiety regarding the pro-
cess. In addition to stress on the child, this complicates the
anesthetic induction and puts considerable distress on the 3 & ' & 3 & /$ & 4
parents. It may also lead to emergence agitation and post-
operative behavioral issues. Understanding the causative Ghazal, Elizabeth A.; Mason, Linda J.; Cotè, Charles J. “Preoperative
factors for preoperative anxiety will help identify those at Evaluation, Premedication, and Induction of Anesthesia” in Cote
CJ, Lerman J, Todres ID, eds. A Practice of Anesthesia for Infants and
risk and guide treatment. Children younger than 7  years Children. 4th ed. Philadelphia, PA: Saunders Elsevier; 2009:39–46.
generally experience more anxiety than older children, as Davidson, Andrew; Howard, Kelly; Browne, William; Harbre, Walid;
the older children tend to develop coping mechanisms to Lopez, Ursula. “Preoperative Evaluation and Preparation, Anxiety,
deal with the anxiety. Several studies have demonstrated Awareness, and Behavior Change” in Gregory GA, Andropoulos
DB, eds. Gregory’s Pediatric Anesthesia. 5th ed. Hoboken,
that shy, withdrawn children will experience more anxiety NJ: Blackwell; 2011:283–9.
than other children their age. This makes it important to Kain ZN, Mayes LC, Caramico LA. Preoperative preparation in chil-
observe the child and question the parents regarding the dren: a cross-sectional study. J Clin Anesth. 1996;8(6):508–14.
child’s personality and reactions to stressful situations. Kogan A, Katz J, Efrat R, Eidelman LA. Premedication with mid-
azolam in young children: a comparison of four routes of adminis-
A  history of a negative experience with anesthesia or sur- tration. Paediatr Anaesth. 2002;12:685–9.
gery will likely influence the preoperative anxiety as well, Tan L, Meakin GH. Anaesthesia for the uncooperative child. Contin
making it important to inquire about past anesthetic Educ Anaesth Crit Care Pain. 2010;10(2):48–52.
experiences.
Treatment of preoperative anxiety is typically mul-
12 . A N S W E R : B
timodal and should be tailored to the child. Research on
the subject has been done for years, but no solid guidelines Parental presence during anesthesia has long been a topic of
exist for treatment. Preoperative education (i.e., pamphlets, controversy. The benefits of parental presence are thought
hospital tours, role playing) completed in the days before to minimize the need for premedication and to avoid the
surgery is effective, but it must be timed well. One author trauma and struggling that comes with separation from
reported that children who underwent preoperative the parents. In addition, there may be more controversial

   r  $ - * / * $ " -  4 $ * & / $ &  0 ' ु " / & 4 5 ) & 4 * "


benefits like decreasing the child’s anxiety as well as the SVC syndrome, orthopnea, and/or pleural effusions.
long-term stigma that is associated with surgery and anes- Patients should then undergo testing, which would include
thesia. It is clear that multiple studies have proven that computed tomography (CT), echocardiography, and PFT
parents and children generally prefer to be together during test when feasible. Vascular and/or airway compression
procedures, but this does not necessarily make it a desirable detected on a CT is associated with a high incidence of
situation or one that has improved patient outcomes. serious complication during the induction of anesthesia.
If parental presence is allowed, the patient and family Echocardiogram detects the compression of the SVC and
must be counseled during the preoperative evaluation about the pulmonary outflow tract, while PFTs detect evidence of
the induction of anesthesia and how they can comfort their the dynamic compression of the airways.
child. It must be stressed that being present is not a right Substantial effort should be made to obtain a tissue
but a privilege and the parent may be removed at any time diagnosis using minimally invasive techniques such as
if there is new or additional risk to the child. It is crucial lymph node biopsy or bone marrow biopsy, so that gen-
to explain specific events during the process to decrease eral anesthesia can be avoided. If obvious signs of vascular
the anxiety parents may feel during the induction. After and/or airway compromise exist, one must consider per-
explaining to the parents what they might see during induc- forming the mediastinoscopy under local, initiating chemo/
tion, it is not unusual for the parents to no longer wish to be radiation to shrink the tumor prior to induction or giving a
present during induction. At no time should the parents be short dose of corticosteroid therapy, which can also shrink
coerced or forced to be present in the operating room. the size of the tumor. These treatments are not without con-
sequence, however, and may actually make it more difficult
to obtain an adequate amount of tissue for a diagnosis; thus,
K E Y FAC T S
the oncologist may choose to avoid such an option.
r The benefits of parental presence during induction over
premedication is still a topic of debate.
K E Y FAC T S
r It is crucial to explain specific events during the
induction process that are normal to decrease the r An AMM can be primary or metastatic, with the most
anxiety parents may feel during the induction. common being lymphoblastic lymphoma and Hodgkin’s
lymphoma.
r When evaluating a patient with an AMM, you should
3 & ' & 3 & /$ & look for signs and symptoms of vascular and/or
airway compromise, which may include dyspnea, pain,
Kain, Zeev N.; MacLaren, Jill; Maes, Linda C. “Perioperative Behavior coughing, SVC syndrome, orthopnea, and/or pleural
Stress in Children” in Cote CJ, Lerman J, Todres ID, eds. A
Practice of Anesthesia for Infants and Children. 4th ed. Philadelphia,
effusions.
PA: Saunders Elsevier; 2009:2. r If obvious signs of vascular and/or airway compromise
exist, one must consider doing the mediastinoscopy
under local, initiating chemotherapy or radiation to
13 . A N S W E R : D shrink the tumor prior to induction or giving a short
This patient is exhibiting signs of SVC syndrome such as dose of corticosteroid therapy, which can also shrink the
distended neck veins, facial swelling, and dyspnea. The size of the tumor.
underlying pathology, however, is vascular congestion
resulting from compression of the SVC by the mass. This is
one of the many unique anesthetic challenges that anterior 3 & ' & 3 & /$ &
mediastinal masses can create for anesthesiologists and thus Hammer, Gregory B. “Anesthesia for Thoracic Surgery” in Cote CJ,
they must be worked up in a logical manner. Lerman J, Todres ID, eds. A Practice of Anesthesia for Infants and
An anterior mediastinal mass (AMM) can be primary Children. 4th ed. Philadelphia, PA: Saunders Elsevier; 2009:290–2.
or metastatic, with the most common being lymphoblastic
lymphoma and Hodgkin’s lymphoma. Other less com-
14 . A N S W E R : C
mon tumors that may be the cause include teratomas, thy-
momas, parathyroid, mesenchymal, and thyroid tumors. In Patients with anterior mediastinal mass (AMM) can have
addition, because non-Hodgkin’s lymphoblastic lymphoma primary or metastatic disease, with the most common
has a 12-hour doubling time, there often is a rush to obtain being lymphoblastic lymphoma and Hodgkin’s lym-
a tissue diagnosis and to begin treatment. phoma. Other less common tumors that may be the cause
When evaluating a patient with an AMM, one should of the patient’s pathology include teratomas, thymomas,
look for signs and symptoms of vascular and/or airway parathyroid, mesenchymal, and thyroid tumors. In addi-
compromise, which may include dyspnea, pain, coughing, tion, because non-Hodgkin’s lymphoblastic lymphoma has

    & 7" - 6"5 * 0 /  0 ' ु $ 0 & 9 * 4 5 * / (  % * 4 & " 4 &  r  


a 12-hour doubling time, there often is a rush to obtain a induction is performed with maintenance of
tissue diagnosis and to begin treatment. spontaneous respiration.
When evaluating a patient with an AMM, one should
look for signs and symptoms of vascular and/or airway
compromise, which may include dyspnea, pain, cough- 3 & ' & 3 & /$ &
ing, SVC syndrome, orthopnea, and/or pleural effusions.
Patients should then undergo testing, which would include Hammer, Gregory B. “Anesthesia for Thoracic Surgery” in Cote CJ,
Lerman J, Todres ID, eds. A Practice of Anesthesia for Infants and
CT, echocardiography, and pulmonary function tests when Children. 4th ed. Philadelphia, PA: Saunders Elsevier; 2009:290–2.
feasible. Vascular and/or airway compression detected on a
CT is associated with a high incidence of serious complica-
tions during the induction of anesthesia. Echocardiogram 15 . A N S W E R : A
detects the compression of the SVC and the pulmonary Latex allergy is extremely common and tends to develop
outflow tract, while PFTs detect evidence of the dynamic after repeated exposures to latex through urinary cath-
compression of the airways. eterizations, surgical gloves, and other products found in
Even if a patient has undergone testing and shows no the hospital. While there has clearly been a more intensive
evidence of vascular and/or airway compromise, severe air- effort at recognizing the significance of the problem and
way and circulatory collapse may still occur with the induc- thus more products have become latex-free, latex still exists
tion of anesthesia. In light of this fact, it is recommended in many products and some facilities are unable to accom-
that an inhalational induction be performed with main- modate latex-free cases.
tenance of spontaneous respiration. In addition, the use The first step in avoiding such a reaction is taking an
of continuous positive airway helps maintain functional adequate history and in particular looking for reactions
residual capacity lost during the induction of anesthesia and with exposure to latex or rubber (toy balloon). Nearly 50%
may help keep the airway from collapsing. Elevating the of patients with a latex allergy will have their first reac-
patient’s head to 30–45 degrees may decrease the deleteri- tion at the dentist’s office, where there is a tendency to use
ous effects of supine positioning. Neuromuscular blockers more latex-containing equipment. Finally, one should be
decrease chest wall tone and should be avoided because cognizant of the link between spina bifida patients and an
this can increase the chance of airway compromise. Finally, increased incidence of latex allergy, as with this patient.
some have advocated using heliox to decrease the airway The presentation of latex allergies is extremely variable.
resistance and to promote laminar flow. Some patients manifest with a simple contact reaction (type
One must be prepared for the chance of airway/vascular 4), while others can manifest with anaphylaxis. These
compromise should the event arise. In circumstances where patients can manifest with variable degrees of rash, bron-
this has occurred, quick repositioning to a prone or lat- chospasm, and/or circulatory collapse. This reaction is due
eral position can be lifesaving. In a situation where airway to a type 1 hypersensitivity reaction.
collapse is the main issue, placing a rigid bronchoscope Treatment of an anaphylactic reaction involves quickly
distal to the point of obstruction may also be necessary. recognizing the problem and washing out the source of
Finally, in catastrophic situations, it has been advocated to contamination. The patient should be given 100% oxy-
perform a median sternotomy and initiate cardiopulmo- gen and volume loaded with a balanced salt solution
nary bypass. Cardiopulmonary bypass would, however, (10–20 mL/kg) with additional boluses given to main-
take some time to initiate and would be impractical unless tain blood pressure. Epinephrine should be given as soon
access for partial bypass was done prior to the induction of as possible at 0.01–0.02 mg/kg and additional vasoactive
anesthesia. medications used to maintain blood pressure. In addition,
beta-agonists can be used to relieve bronchoconstric-
tion, corticosteroids to diminish mediator formation and
K E Y FAC T S
release, and antihistamines (H1 and H2 antagonists) to
r Patients with AMM can have primary or metastatic block the action of histamine.
disease, with the most common being lymphoblastic
lymphoma and Hodgkin’s lymphoma.
K E Y FAC T S
r When evaluating a patient with an AMM, you should
look for signs and symptoms of vascular and/or r Nearly 50% of patients with a latex allergy will have
airway compromise, which may include dyspnea, pain, their first reaction at the dentist’s, where there is a
coughing, SVC syndrome, orthopnea, and/or pleural tendency to use more latex-containing equipment.
effusions. r There is an association between spina bifida and an
r Severe airway and circulatory collapse can still occur increased incidence of latex allergy.
with induction. It is recommended that an inhalational r Anaphylaxis is a type 1 hypersensitivity reaction.

   r  $ - * / * $ " -  4 $ * & / $ &  0 ' ु " / & 4 5 ) & 4 * "


3 & ' & 3 & /$ & 3 & ' & 3 & /$ & 4

Everett, Lucinda L.; Fuzaylov, Gennadiy; Todres, I. David. “Pediatric Cote CJ. Pediatric anesthesia. In: Miller RD, Eriksson LI, Fleisher LA,
Emergencies” in Cote CJ, Lerman J, Todres ID, eds. A Practice Wiener-Kronish JP, Young WL, eds. Miller’s Anesthesia. 7th ed.
of Anesthesia for Infants and Children. 4th ed. Philadelphia, Orlando, FL: Churchill Livingstone; 2009:xx–xx.
PA: Saunders Elsevier; 2009:779. Insoft, Robert M.; Todres, I.  David. “Growth and Development” in
Cote CJ, Lerman J, Todres ID, eds. A Practice of Anesthesia for
Infants and Children. 4th ed. Philadelphia, PA: Saunders Elsevier;
2009:21.
16 . A N S W E R : B
Gastroesophageal reflux disease (GERD) is common in
17. A N S W E R : B
neonates and preterm infants, with up to 40% of new-
borns regurgitating their food. This is thought to be due This question tackles many of the issues that are associ-
at least partially to the inability to coordinate swallowing ated with patients with GER in the peri-induction period.
and respiration, which does not fully develop until about Gastroesophageal reflux (GER) is common in neonates and
4–5 months gestational age. In addition, lower esophageal preterm infants, with up to 40% of newborns regurgitating
sphincter pressures are diminished and take 3–6 weeks to their food. In general, infants with the diagnosis are other-
achieve adult levels. Neonatal duodenal maturity does wise healthy but may occasionally have concomitant devel-
not occur until 29–32 weeks and may be a relevant con- opmental abnormalities. This is a more common scenario.
tributor in the preterm infant. Finally, CNS changes may It is also important to note that the risk of aspiration on
also contribute. induction is increased in all children that are developmen-
GERD must be initially differentiated from develop- tally delayed, have had a previous esophageal surgery,
mental abnormalities such as pyloric stenosis, duodenal have had a difficult airway, were obese, underwent trau-
stenosis, annular pancreas, malrotation, tracheoesophageal matic injury, and have coexisting GER.
fistula, and esophageal atresia. In general, developmental Whether the patient has GER or not, it is important to
abnormalities usually present within the first 24–36 hours recognize the recommended fasting periods for elective sur-
of life—upper intestinal abnormalities generally present as gery in healthy patients and their limitations. Keep in mind
recurrent vomiting and regurgitation, while lower abdomi- that the goal of fasting is to reduce gastric volume and that
nal abnormalities present with abdominal distention and because children are often induced via inhalation their risk
failure to pass meconium. The exception would be pyloric for aspiration is often protracted. Table 20.1 shows recom-
stenosis, which typically presents in the third week of life. mendations that are found in ASA practice guidelines.
The diagnosis of GERD often takes an accurate history and Premedication for GERD and the prevention of aspi-
physical examination as well as one or more of the follow- ration pneumonitis classically involves antacids, H2 antago-
ing tests: (1) barium swallow, (2) PH probe, (3) upper GI nists, proton pump inhibitors, and gastrointestinal motility
endoscopy, (4) gastric emptying study. agents. Antacids work by increasing pH, and nonparticulate
GERD in the infant and neonate typically presents antacids should be used such as sodium citrate. Particulate
with irritability, persistent vomiting, failure to thrive, antacids can cause a severe pneumonitis if aspirated and
and in severe cases hematemesis and anemia and can be should be avoided. H2 antagonists work by inhibiting
occasionally complicated with stricture formation. In addi- acid secretion in the gastric mucosa leading to increased
tion, reflux into the pulmonary system may lead to recur- pH and decreased residual volume. Proton pump inhibi-
rent pneumonia, irritability, wheezing, and stridor. tors work in a similar manner; however, no PPI has been
approved in children below the age of 1  year. It is impor-
tant to recognize that H2 blockers and PPI need at least
K E Y FAC T S 30 minutes to be effective and 45 minutes–1 hour for peak
r GERD is common in neonates and preterm infants,
Table 20.1 "4"3 &$0..&/%"5*0/4
with up to 40% of newborns regurgitating their food.
'03ु'"45*/(1&3*0%4'03ु&-&$5*7&
r Developmental abnormalities usually present within
463(&3:
the first 24–36 hours of life—upper intestinal
abnormalities generally present as recurrent vomiting
$MFBST IS
and regurgitation, while lower abdominal abnormalities
#SFBTUNJML IS
present with abdominal distention and failure to pass
meconium. *OGBOUGPSNVMB IS
r GERD typically presents with irritability, persistent 4PMJET IS
vomiting, failure to thrive, and in severe cases SOURCE: Reprinted with permission from Cote CJ, Lerman J,
hematemesis and anemia, and can lead to recurrent Todres ID, eds. A Practice of Anesthesia for Infants and Children.
4th ed. Philadelphia, PA: Saunders Elsevier; 2009:167.
pneumonia, irritability, wheezing, and stridor.

    & 7" - 6"5 * 0 /  0 ' ु $ 0 & 9 * 4 5 * / (  % * 4 & " 4 &  r   


Table 20.2 13 &.&%*$"5*0/'03 cell stabilizers are two final classes of agents commonly
("4530&401)"(&"-3 &'-69"/%5)& encountered in this patient population.
13 &7&/5*0/0'ु"41*3 "5*0/1/&6.0/*5*4 Immunoglobulin E–mediated atopy is a common find-
%04*/(*/5)&1&%*"53*$1016-"5*0/ ing among asthmatics. Atopic asthmatics tend to have less
well-controlled disease as compared to nonatopic patients.
#JDJUSB N- Nonatopic asthmatics may, in fact, have a subtly different
.FUPDMPQSBNJEF QSPLJOFUJD
mNHLH pathophysiologic process from atopic asthmatics evidenced
3BOJUJEJOF )BOUBHPOJTU
mNHLH by the fact that nonatopic patients exhibit reduction of
'BNPUJEJOF )BOUBHPOJTU
mNHLH symptoms with age.
Asthmatics present with a variety of symptoms, ranging
from subtle chest tightness to respiratory failure. Patient
effect. Promotility agents such as metoclopramide work presentation may include air hunger, wheezing, acces-
by increasing lower esophageal tone, relaxing the pyloric sory muscle use, coughing, pneumothorax, mental status
sphincter, and promoting gastric motility while decreasing change, and prolonged expiration among other associated
gastric volume. Metoclopramide works 1–2 minutes after symptoms. Asthma, like other obstructive respiratory dis-
IV administration. Table 20.2 shows the recommended eases, does not manifest as prolonged inspiration.
dosing in the pediatric population.
K E Y FAC T S

3 & ' & 3 & /$ & 4 r Asthma is a complicated disease that involves chronic
inflammation and lower airway reactivity.
Cote CJ. Pediatric anesthesia. In: Miller RD, Eriksson LI, Fleisher LA, r It commonly presents with air hunger, wheezing,
Wiener-Kronish JP, Young WL, eds. Miller’s Anesthesia. 7th ed. accessory muscle use, coughing, pneumothorax, mental
Orlando, FL: Churchill Livingstone; 2009:xx–xx. status change, and prolonged expiration.
Insoft, Robert M.; Todres, I.  David. “Growth and Development” in
Cote CJ, Lerman J, Todres ID, eds. A Practice of Anesthesia for
r Treatment includes beta-agonists, corticosteroids,
Infants and Children. 4th ed. Philadelphia, PA: Saunders Elsevier; leukotriene inhibitors, and mast cell stabilizers.
2009:532.

18 . A N S W E R : B 3 & ' & 3 & /$ &


Asthma is a complicated disease process that is likely a fam- Firth, Paul G.; Kinane, T.  Bernard. “Essentials of Pulmonology” in
ily of related disease processes which present with a com- Cote CJ, Lerman J, Todres ID, eds. A Practice of Anesthesia for
mon finding of wheezing. Infants and Children. 4th ed. Philadelphia, PA: Saunders Elsevier;
2009:229–32.
Critical to the understanding of asthma is an apprecia-
tion that lower airway reactivity and spasm are merely
components of the overall disease process. Inflammation
19. A N S W E R : C
is a critical component underlying intermittent broncho-
spasm. Chronic inflammation leads to loss of appropri- Infants and children as well as adults are fasted prior to
ate bronchoalveolar interaction and permanent airway operations to decrease gastric volume and minimize the
remodeling. risk of aspiration. The thinking behind this is that in the
Therapy for asthma is directed at reducing under- fasted child the only thing that would remain in the stom-
lying inflammation while controlling acute symptom- ach would be basal gastric secretions. The risk and incidence
atology. Beta agonists as well as ipratropium decrease of aspiration in routine procedures without risk factors is
smooth muscle contraction and thus reduce bronchospasm. small, and it is recommended that in children without addi-
Methylxanthines such as theophylline additionally reduce tional risk factors be fasted for 2 hours after the consump-
bronchospasm but are associated with toxicity and are tion of clear liquids. These recommendations come from
becoming historical rather than practical. Corticosteroids, studies that showed no additional decrement in risk when
both inhaled and systemic, are commonly prescribed and fasting a child greater than 2 hours. Clear liquids include
used in asthmatic patients. These agents reduce swelling water, fruit juices without pulp, carbonated beverages,
and are well tolerated but are associated with hypergly- clear tea, and black coffee. Finally, the gastric residual
cemia, hypertension, and loss of endocrine homeostasis. half-life of clear liquids is approximately 15 minutes, and
It is useful to note that inhaled corticosteroids do induce this information could be used to guide additional fast-
systemic effects as the dose escalates in comparison to ing time for patients with some risk factors. The benefits
patient size. Infants may be at risk for systemic effects of of fasting for 2 hours but not longer include a decreased
inhaled corticosteroids. Leukotriene inhibitors and mast risk for hypoglycemia, hypovolemia, decreased thirst,

   r  $ - * / * $ " -  4 $ * & / $ &  0 ' ु " / & 4 5 ) & 4 * "


Table 20.3 13 &01&3 "5*7&'"45*/( reduce the chance of respiratory complications, this is a
3 &$0..&/%"5*0/4*/ु*/'"/54"/%$)*-%3 &/ practice that may not be compatible with real life and may
even put the anesthesiologist in the position of postponing
$MFBSMJRVJET IPVST surgery that could ultimately break the cycle of infection.
#SFBTUNJML IPVST Noninvasive airway management (mask or LMA) has
*OGBOUGPSNVMB IPVST† demonstrated a benefit in reducing respiratory complica-
4PMJET GBUUZPSGSJFEGPPET
IPVST tions in the setting of URI. Interestingly, positive-pressure
ventilation alone appears to increase the risk of respiratory
*Include only fluids without pulp, clear tea, or coffee without milk products. complication in such patients, placing a premium on spon-

Some centers allow plain toast (no dairy products) up to 6 hours prior to taneous ventilation.
induction.
While pharmacologic intervention is tempting for the
SOURCE: Reprinted with permission from Warner MA, Caplan RA,
Epstein B. Practice guidelines for preoperative fasting and the use of
patient with a URI, few medications have demonstrated a
pharmacologic agents to reduce the risk of pulmonary aspiration: application positive effect. Bronchodilators given preoperatively in oth-
to healthy patients undergoing elective procedures: a report by the American erwise healthy children having minor surgery have shown
Society of Anesthesiologist Task Force on Preoperative Fasting. Anesthesiology.
1999;90:896–906. no protective effect. Anticholinergics have shown little
added benefit in this arena as well. Humidification of gasses
as well as adequate vascular hydration have some benefit in
decreased hunger, decreased risk for hypotension on reducing respiratory complications.
induction, and improved child cooperation. The nature of the surgery is distinctly important in
Breast milk has a high lipid content, which delays gas- the frequency of respiratory complications in children
tric emptying and can cause significant pulmonary injury with a URI. Head and neck surgery as well as thoracic or
if aspirated. Furthermore, studies have shown that gastric intra-abdominal surgery is associated with more frequent
emptying times for breast milk when compared to clear respiratory complications in affected children.
liquids are substantially greater. In addition, gastric empty- A discussion of degree of illness is warranted. Viral
ing times for formula are substantially greater than breast respiratory illness appears to be less severe than bacterial
milk. It is because of these studies that the ASA recom- illness. Discerning the difference between these two states
mended guidelines, as shown in Table 20.3. is at times difficult. Generally speaking, presence of a pro-
ductive, moist cough is associated with more severe bac-
terial infection. Additionally, fever greater than 38.5°C,
3 & ' & 3 & /$ & wheezing or rales on lung exam, and behavioral changes
are associated with severe disease. In addition, formerly pre-
Ghazal, Elizabeth A.; Mason, Linda J.; Cotè, Charles J. “Preoperative mature children and young (less than 1 year) children tend
Evaluation, Premedication, and Induction of Anesthesia” in Cote to have more severe responses to URIs and should be con-
CJ, Lerman J, Todres ID, eds. A Practice of Anesthesia for Infants and sidered separately from the otherwise healthy children dis-
Children. 4th ed. Philadelphia, PA: Saunders Elsevier; 2009:37–8.
cussed thus far. Household smoking also appears to worsen
the outcome in affected patients.
2 0 . A N S W E R : B
The issues surrounding the child with upper respiratory K E Y FAC T S
infection have best been described as vexing. While man-
r Noninvasive airway management (mask or LMA)
agement of children on the extreme ends of the infectious
has demonstrated a benefit in reducing respiratory
spectrum is fairly straightforward from the surgical plan-
complications in the setting of a URI.
ning perspective, the majority of children present with
r Positive-pressure ventilation alone appears to increase
nuanced symptoms such as rhinorrhea.
the risk of respiratory complication in such patients,
Postponement of surgery is an option in children with
placing a premium on spontaneous ventilation.
any infectious symptomatology. Such a stance is, however,
r Fever greater than 38.5°C, productive cough, wheezing
rather restrictive. Given that 20% to 30% of children will
or rales on lung exam, and behavioral changes are
present with upper respiratory infection (URI) symptoms,
associated with severe disease.
the distinct possibility of another cold occurring during
convalescence is present. Layering on top of this informa-
tion, the very nature of many pediatric surgeries is aimed at
3 & ' & 3 & /$ &
reducing the frequency and severity of infection. Examples
of such surgery include adenoidectomy, tonsillectomy, and Firth, Paul G.; Kinane, T. Bernard. “Essentials of Pulmonology” in Cote
myringotomy. While postponing surgery 7 weeks (the time CJ, Lerman J, Todres ID, eds. A Practice of Anesthesia for Infants and
frame required for spirometry studies to normalize) will Children. 4th ed. Philadelphia, PA: Saunders Elsevier; 2009:226–9.

    & 7" - 6"5 * 0 /  0 ' ु $ 0 & 9 * 4 5 * / (  % * 4 & " 4 &  r 


21. A N S W E R : A lack of decision-making ability. Between the ages of 7 and
14 years, children may develop decision-making skills, but it
Anesthetic risk for children differs from that of adult patients.
is unlikely that a patient in this age range will reliably dem-
In fact, adverse events (major and minor collectively) have
onstrate such skill. Children above the age of 14 years may
been reported to occur in 35% of pediatric cases as compared
reliably hold adequate decision-making skills. It is critical
to 17% in adult anesthesia. This difference in risk is multi-
that the practitioner not confuse this ability with the abil-
faceted and includes patient characteristics as well as surgical
ity to engage in informed consent because children of this
characteristics. Children present a number of anatomic and
age lack cognitive and emotional skills required to fully
physiologic challenges for the anesthesiologist. These chal-
participate in the process of consent.
lenges generally become more pronounced with decreasing
There are four fundamental steps to informed con-
patient age and size. Empirically, children 1  month of age
sent. First, the patient or surrogate (often a parent) must be
and younger have been reported to have the highest rate of
supplied adequate information to understand both his or
adverse event. Importantly, the same database that yielded
her medical condition and the procedure planned for him
this finding also reported that children under 1  month of
or her. Second, the practitioner must assess the patient or
age underwent cardiac and major vascular surgery more fre-
surrogate for understanding of the information laid out
quently than did children in other age groups.
in the first step. Third, the practitioner must assess the
Particular organ systems appear to be more prone to
patient/surrogate for capacity (cognitive, emotional, and
adverse event than others. In a second database, respira-
decision making) to engage in consent. Fourth, the practi-
tory adverse events accounted for 53% of all intraoperative
tioner must deliver an assurance that the patient/surrogate
events. Infants, patients undergoing ENT surgery, patients
may make the decision free of coercion.
in whom the trachea was intubated, and patients with
Pediatric informed consent is further complicated by
ASAPS scores of 3 and above were more likely to have a
several unique situations. First, the surrogate may (and
respiratory adverse event. Cardiac adverse events accounted
often does) ask the physician what the physician would do
for 12.5% of all such events. Nausea occurred in 6% of cases
in a given setting. This question is fundamentally flawed
and appeared (in both databases) to occur in older children.
because the child simply is not the physician’s. Any declara-
Cohen reports that one third of 5-year-old patients experi-
tive answer to this question is simply a guess. No physician
enced vomiting in recovery.
knows his or her patient well enough to claim to know or
understand all the complexity of the parent–child rela-
K E Y FAC T S tionship. The best answer to this question will be one that
addresses the goals that the family is attempting to
r Adverse events have been reported to occur in 35% of
achieve and the values they hold. Second, some children
pediatric cases, as compared to 17% in adult cases.
obtain emancipated minor status via the legal system. These
r Children 1 month of age and younger have been
individuals are typically adolescent and either married,
reported to have the highest rate of adverse event.
parents, in the military, or economically independent
r Most adverse events are respiratory (53%) or cardiac
of their parents for any of a variety of reasons. Each state
(12.5%) in nature.
handles these individuals and their rights uniquely, but
in general they are recognized as holding the right to par-
3 & ' & 3 & /$ & 4 ticipate in informed consent. Third, there exist religious
and cultural differences between patients and families. Of
Cohen MM, Cameron CB, Duncan PG. Pediatric anesthesia mor- common concern are children of Jehovah’s Witnesses.
bidity and mortality in the perioperative period. Anesth Anelg. Without great detail, such families generally refuse blood
1990;70(2):160–7.
Murat I, Constant I, Maud’Huy H. Perioperative anesthetic morbid-
transfusion. Practitioners may or may not agree with this
ity in children:  a database of 24,165 anesthetics over a 30-month choice. Furthermore, it is debatable whether the child must
period. Pediatr Anesth. 2004;14:158–66. be treated as an informed practitioner of the religion or cul-
tural practice. Each clinician must decide on a case-by-case
basis whether to support this decision. Consultation with
22 . A N S W E R : C
representatives of the legal system prior to surgery may be
Informed consent, a complicated concept for adult patients, wise prior to surgery if a conflict exists between the practi-
is significantly more complicated in the arena of pediatrics tioner and the family/patient regarding such religious and
because children are widely deemed to lack the cognitive, cultural norms and their impact on medical care.
emotional, and decision-making skills required to partici-
pate alone through the consent process. Assent, however,
K E Y FAC T S
is obtainable according to the child’s capability. A  “rule
of sevens” aids in this assent process. Children younger r Children above the age of 14 years may reliably hold
than 7 years of age are typically incapable of assent due to a adequate decision-making skills, but they often lack

   r  $ - * / * $ " -  4 $ * & / $ &  0 ' ु " / & 4 5 ) & 4 * "


the cognitive and emotional skills required to fully Given that the patient described in this question has no
participate in the process of consent. comorbidities and is undergoing a benign surgical procedure
r An emancipated minor (adolescents that are married, with little anticipated blood loss, no testing is warranted.
parents, in the military, or economically independent) is
generally recognized as holding the right to participate
in informed consent. 3 & ' & 3 & /$ & 4
r Consultation with representatives of the legal system
prior to surgery may be wise if a conflict exists between Ghazal, Elizabeth A.; Mason, Linda J.; Cotè, Charles J. “Preoperative
Evaluation, Premedication, and Induction of Anesthesia” in Cote
the practitioner and the family/patient regarding CJ, Lerman J, Todres ID, eds. A Practice of Anesthesia for Infants and
such religious and cultural norms and their impact on Children. 4th ed. Philadelphia, PA: Saunders Elsevier; 2009:43–4.
medical care. Miller-Hance, Wanda C. “Anesthesia for Noncardiac Surgery in
Children with Congenital Heart Disease” in Cote CJ, Lerman J,
Todres ID, eds. A Practice of Anesthesia for Infants and Children. 4th
ed. Philadelphia, PA: Saunders Elsevier; 2009:467–8.
3 & ' & 3 & /$ &

Waisel, David B. “Ethical Issues in Pediatric Anesthesiology” in Cote 2 4 . A N S W E R : C


CJ, Lerman J, Todres ID, eds. A Practice of Anesthesia for Infants and
Children. 4th ed. Philadelphia, PA: Saunders Elsevier; 2009:73–6. Parent presence on induction of anesthesia (PPI) is a process
of variable popularity that holds many potential advantages
23 . A N S W E R : D as well as disadvantages. Each practitioner and institu-
tion must carefully develop and adopt a position regard-
Lab testing and other studies such as EKG and lung func- ing the practice of PPI; no national standard is available.
tion tests are certainly useful in aiding the assessment of Commonly, patient age comes under question in develop-
the appropriate patient. Metabolic and electrolyte testing is ment of such a plan. While it is intuitive that adult patients
useful in patients with disorders of the cardiac, gastrointes- often do not require or request PPI, unique patients may
tinal, and genitourinary systems as well as patients receiving benefit from PPI. Such patients may be developmentally
psychiatric medications. Blood counts are useful in the delayed or even demented, calling into question the appro-
setting of significant blood loss and hematologic disor- priateness of more broad practices such as family present
ders as well as children with oncologic disorders. More induction for the elderly.
important than memorizing tests is that the practitioner Both benefits and challenges exist in regard to PPI.
arrive at a point where he or she understands why a test is Potential benefits include a lessened requirement for preop-
being used and when it is warranted. erative sedation medications, less frequent emergence delir-
A fundamental understanding of the patient’s current ium, and a more benign induction process. Importantly,
physiology and the anticipated surgical procedure are essen- data regarding these benefits are mixed, and it cannot be
tial in driving preoperative assessment. A  useful starting conclusively stated that these benefits in fact exist across all
point is to obtain laboratory studies that will either (a) yield patients. Disadvantages of PPI include requirement of addi-
information regarding the patient’s current status, (b) alter tional staff, possible disruption of the operating room and
further assessment (such as blood typing and screening for sterile area, and harm either to or inflicted by the parent.
anemic patients), or (c)  alter the patient’s surgical plan- The parent, it must be recognized, is not an inert cog in this
ning (such as electrolyte testing for dehydrated patients). machine but brings with him or her a degree of unpredict-
A discussion regarding echocardiography in particular is ability. The parent must be counseled regarding what will be
warranted. In the context of congenital heart disease (CHD) seen in the operating room and what each individual’s role
it is often useful to obtain a recent echocardiogram. Given is in the event of an emergency, parent included. Beyond the
that children with CHD undergoing surgery often mani- possibility that the parent may disrupt the operative envi-
fest symptoms consistent with cardiac failure, visualization ronment, the operative environment may harm the parent!
of current anatomy and function provides the practitioner Data exist describing elevations in heart rate, blood pres-
a wealth of information. Cardiac failure in children may sure, and skin conductance in parents participating in PPI.
manifest much more obscurely than in adults. Simple failure While no harm has been reported to parents during PPI, it
to match peers in growth raises many questions regarding a is important that practitioners fully recognize the gambit of
neonate’s cardiac status. Rarely is the same true for adults. outcomes that may develop in the context of PPI.
Given that cardiac dysfunction is difficult to diagnose clini-
cally in pediatrics, empiric evaluation via echocardiogra-
phy is frequently warranted. Children with cardiac disease 3 & ' & 3 & /$ &
often have undergone repeated echocardiography, which
may be useful in assessment. A discussion with the patient’s Kain, Zeev N.; MacLaren, Jill; Maes, Linda C. “Perioperative Behavior
cardiologist is often helpful to this end. Stress in Children” in Cote CJ, Lerman J, Todres ID, eds. A Practice

    & 7" - 6"5 * 0 /  0 ' ु $ 0 & 9 * 4 5 * / (  % * 4 & " 4 &  r   


of Anesthesia for Infants and Children. 4th ed. Philadelphia, Children who have been hospitalized in the past are a
PA: Saunders Elsevier; 2009:28–31. unique population. A trend toward an enhanced emotional
response to hospitalization is present in children who
25 . A N S W E R : D have been hospitalized in the past. A prior negative medi-
cal experience additionally promotes anxiety. Appropriate
Psychosocial preparation of the pediatric patient is multi- preparation for such patients may be best targeted at spe-
faceted in regard to both breadth and patient applicability. cific experiences.
Common techniques aim at creating trusting relationships
with health care providers, using age-appropriate terminol-
ogy, and developing coping skills. 3 & ' & 3 & /$ &
Of interest, patient age is an important consideration
in psychosocial preparation programs. The fundamental Ghazal, Elizabeth A.; Mason, Linda J.; Cotè, Charles J. “Preoperative
consideration appears to be the child’s ability to discern Evaluation, Premedication, and Induction of Anesthesia” in
between fantasy and reality. At approximately 3 years of Cote CJ, Lerman J, Todres ID, eds. A Practice of Anesthesia for
Infants and Children. 4th ed. Philadelphia, PA: Saunders Elsevier;
age, children become able to differentiate between these 2009:39–41.
two states. With this development, the child becomes able
to benefit from simulation meant to prepare him or her for
upcoming surgery. As such, children younger than 3 years 2 6 . A N S W E R : B
may not benefit from such programs.
Older children demonstrate an interesting nuance to Cerebral palsy, a static encephalopathy, is the most com-
preparatory programs as well. Children aged 6  years and mon childhood movement disorder (Table 20.4). Cerebral
older have demonstrated an increase in anxiety leading up palsy has an incidence of 1/7000 births and a prevalence
to the date of preoperative preparation. In fact, this anxi- of 1/5000. Often there are no identifiable risk factors, and
ety peaked in the days surrounding such preparation and the diagnosis is made when a patient exhibits delayed motor
fell below baseline levels 5 days following preparation. development. Despite the static nature of neurologic dam-
Given this trend, it is critical that older children be sched- age, the clinical picture for this disease state is variable
uled for preparation a week in advance of surgery. and may change. It is thought that insults to the imma-
Regarding a parent’s influence on a child’s level of anxi- ture CNS or congenital anomalies cause cerebral palsy, as
ety, it is not surprising that the values positively correlate. birth asphyxiation, prematurity, and intrauterine growth
Anxious parents beget anxious children. Preparation must retardation are known associations. Approximately 75%
include the family in addition to the patient. Furthermore, of cases involve in utero insults to the immature CNS,
ideal preparation will clearly convey to the parent how 10% of cases occur during birth, and the remaining 15%
directly their state of mind influences that of their child. of cases occur shortly after birth. Despite immature CNS

Table 20.4 $&3 &#3 "-ु1"-4:

TYPE/ETIOLOGY MOTOR DEFICIT DISTR IBUTION COMPLICATIONS

Hypotonic -PXBYJBMुUPOF %JĈVTF -FBSOJOHEJTBCJMJUZ


4ZOESPNJD 7BSJBCMFMJNCुUPOF $POUSBDUVSFT
%ZTHFOFTJT %FFQUFOEPOSFĔFYFTVTVBMMZ &QJMFQTZ
*OTVMUुIZQPYJBJTDIFNJB JODSFBTFE 'FFEJOHEZTGVODUJPO
)FBSJOHWJTJPOJNQBJSNFOU
3FTQJSBUPSZJOGFDUJPOT
Spastic *ODSFBTFEUPOFुQZSBNJEBMुUZQF .POPQBSFTJT
*OTVMUुIZQPYJBJTDIFNJB  *ODSFBTFEEFFQUFOEPOSFĔFYFT %JQBSFTJT
WBTDVMBS )FNJQBSFTJT
5SJQBSFTJT
5FUSBQBSFTJT
Choreoathetoid *OWPMVOUBSZNPWFNFOUुPĕFO .BZCFEJĈVTF UFUSBQBSFTJT
PS )FBSJOHJNQBJSNFOU
*OTVMUुIZQPYJBJTDIFNJB BNJYUVSFPGDIPSFBBUIFUPTJT DPOđOFEUPPOFPSNPSFुMJNCT $POUSBDUVSFT
OFPOBUBMIZQFSCJMJSVCJOFNJB EZTUPOJB 0ĕFODPFYJTUTXJUITQBTUJDJUZ *OUFMMFDUPĕFONBJOUBJOFE
NFUBCPMJD
Ataxic "UBYJBVTVBMMZHFOFSBMJ[FE .BZCFEJĈVTFCVUPĕFOBTTPDJBUFE 'FX NBZCFNJME
$FSFCSBMEZTHFOFTJT USVODBMBOEुMJNC XJUIEJQBSFTJT
.BZDPFYJTUXJUITQBTUJDJUZ
SOURCE: Reprinted with permission from: Cote CJ, Lerman J, Todres ID, eds. A Practice of Anesthesia for Infants and Children. 4th ed. Philadelphia,
PA: Saunders Elsevier; 2009:493.

   r  $ - * / * $ " -  4 $ * & / $ &  0 ' ु " / & 4 5 ) & 4 * "


injury, cognitive impairment is not a consistent feature of Treatment for CF focuses on nutritional reple-
cerebral palsy. tion with high-calorie diets, continuation of pancreatic
Cerebral palsy is classified according to its severity, dis- enzyme replacement, and fat-soluble vitamin supple-
tribution, and nature of motor deficit. The four major classi- ments; and relief of airway obstruction, including pre- and
fications of cerebral palsy are spastic, athetoid or dyskinetic, postoperative bronchodilator use, incentive spirometry,
ataxic, and mixed. Without the normal gravitational postural drainage, and pathogen-specific antibiotics
muscular stresses, abnormal skeletal muscle development for lung infection. Other areas currently being evaluated
ensues, and patients with cerebral palsy often require multi- for treatment of CF include anti-inflammatory medication
ple orthopedic surgeries to loosen tight muscles and joint and lung transplant. Answer A is incorrect because preop-
contractures, and to straighten abnormal long bone twists erative bronchodilators should be continued to optimize
and kyphoscoliosis. postoperative outcomes. Answer B is incorrect because
Although severe scoliosis compromises respiratory pathogen-specific antibiotics should be administered. In
reserve, idiopathic scoliosis by itself is not strongly associ- addition, prophylactic broad-spectrum antibiotics may har-
ated with impaired oromotor function or recurrent aspira- bor multidrug resistance. Answer D is incorrect because
tion pneumonia. postoperative outcomes are optimized when CF patients
Similarly, pulmonary reserve is compromised in continue high-calorie diets, coupled with replacement of
patients with cystic fibrosis (CF), but severe CF does not pancreatic enzymes and fat-soluble vitamin supplements.
affect oromotor function. Rather, severe CF disrupts elec-
trolyte transport in epithelial cells of sweat ducts, airway,
pancreatic duct, intestinal, biliary tree, and vas deferens. 3 & ' & 3 & /$ & 4
Although Trisomy 21 is associated with duodenal atresia
and generalized hypotonia, severe cerebral palsy is more Firth P, Haver K. “Essentials of Pulmonology” in Cote CJ, Lerman J,
Todres ID, eds. A Practice of Anesthesia for Infants and Children. 4th
often associated with severe seizure disorder, impaired ed. Philadelphia, PA: Saunders Elsevier; 2009:233–5.
bulbar function, and recurrent aspiration pneumonia. Holzman RS, Mancuso TJ, Sethna NF, DiNardo JA. “Respiratory
With severe bulbar dysfunction, some patients with cere- System” in Holzman RS, Mancuso TJ, Sethna NF, DiNardo JA,
bral palsy have difficulties feeding and speaking. Coupled eds. Pediatric Anesthesiology Review. New  York, NY:  Springer;
2010:48–9.
with significant gastroesophageal reflux disease and poor
laryngeal reflexes, patients with cerebral palsy can have
recurrent aspiration pneumonia and subsequent reduced
2 8 . A N S W E R : B
pulmonary reserve.
The best way to prevent latex allergy is to remove latex
from hospitals. To this end, many children’s hospitals are
3 & ' & 3 & /$ & 4 converting to completely latex-free environments. Latex
allergy is IgE mediated, and clinical diagnosis requires a
Crean P, Hicks E. “Essentials of Neurology and Neuromuscular Disease” high index of suspicion.
in Cote CJ, Lerman J, Todres ID, eds. A Practice of Anesthesia for
Infants and Children. 4th ed. Philadelphia, PA: Saunders Elsevier;
The following individuals are at increased risk of ana-
2009:492–3. phylaxis to latex:  children with neural tube defects such
Anderson B, Wilton N. “Orthopedic and Spine Surgery” in Cote CJ, as spina bifida and myelomeningocele, children with
Lerman J, Todres ID, eds. A Practice of Anesthesia for Infants and urinary tract anomalies requiring frequent bladder cath-
Children. 4th ed. Philadelphia, PA: Saunders Elsevier; 2009:651–2.
Urban, MK. “Anesthesia for Orthopedic Surgery” in Miller RD,
eterization, children with an immunologic predisposition,
Eriksson LI, Fleisher L, Wiener-Kronish JP, eds. Miller’s Anesthesia. children undergoing multiple surgical procedures, individ-
7th ed. Philadelphia, PA: Churchill Livingstone; 2010:2248. uals with an allergy to tropical fruits such as banana and
kiwi, and health care workers exposed to latex.
Although it has been demonstrated that pretreatment
27. A N S W E R : C
with antihistamine and corticosteroid can prevent a reac-
Cystic fibrosis (CF) demonstrates autosomal recessive tion to intravenous contrast, these preventative treatments
inheritance and occurs in 1/2000 Caucasian births. The do not reliably prevent anaphylaxis to latex. Thus, Answers
pathophysiology for CF involves chronic infection, A and C are incorrect.
mucus plugging, inflammation, and epithelial injury, If anaphylaxis is clinically suspected, the most impor-
as the mutation in the gene encoding CF transmembrane tant treatment is intravenous epinephrine (1–10 µg/kg)
conductor regulator disrupts electron transport via chlo- to prevent further mast cell degranulation. In addition,
rine channel in various epithelial cells. This renders CF the offending agent should be removed or diluted, 100%
patients to the symptoms of productive cough, hemop- oxygen should be administered, the patient should be intu-
tysis, recurrent respiratory infections, atelectasis, and bated if not already, acute volume loading should occur, and
maldigestion. the patient may require an infusion of epinephrine or other

    & 7" - 6"5 * 0 /  0 ' ु $ 0 & 9 * 4 5 * / (  % * 4 & " 4 &  r   


vasoactive mediations to support the patient’s blood pres- anterior mediastinal mass, this patient may demonstrate
sure. If possible, a blood sample should be sent for mast cell extrathoracic or intrathoracic variable or fixed obstruc-
tryptase test to confirm anaphylaxis, but this is certainly tion, and this would be apparent on flow-volume studies
not the first or most important step in treating anaphylaxis. (see Fig. 20.1).

3 & ' & 3 & /$ & 3 & ' & 3 & /$ & 4

Everett LL, Fuzaylov G, Todres ID. “Pediatric Emergencies” in Cote CJ, Firth PG, Haver KE. “Essentials of Pulmonology” in Cote CJ, Lerman
Lerman J, Todres ID, eds. A Practice of Anesthesia for Infants and J, Todres ID, eds. A Practice of Anesthesia for Infants and Children.
Children. 4th ed. Philadelphia, PA: Saunders Elsevier; 2009:779. 4th ed. Philadelphia, PA: Saunders Elsevier; 2009:227.
Hammer GB. “Anesthesia for Thoracic Surgery” in Cote CJ, Lerman
J, Todres ID, eds. A Practice of Anesthesia for Infants and Children.
29. A N S W E R : D 4th ed. Philadelphia, PA: Saunders Elsevier; 2009:291.

Pediatric anterior mediastinal masses may be primary or


metastatic tumors of the lung, mediastinum, and pleura.
3 0 . A N S W E R : A
Associated signs and symptoms are typically due to air-
way or vascular compression and include stridor, dys- Although potential symptoms of a URI and a potential
pnea, orthopnea, cough, pain, pleural effusion, and SVC difficult airway should alert any pediatric anesthesiologist,
syndrome. This patient demonstrates dyspnea, orthopnea, the cardiovascular assessment of this patient with Williams
cough, and SVC syndrome. syndrome supersedes other perioperative concerns. With
Preoperative CT scan, cardiac echocardiogram, and only rhinorrhea and a dry cough, some would argue that
pulmonary function tests should be obtained for anterior this child does not have an active URI. Micrognathia with
mediastinal masses. However, this question asks which prominent incisors does not necessarily equate with diffi-
preoperative study is most appropriate to detect dynamic cult airway, but it would be prudent to have advanced and
airway compression. A plain chest X-ray does not detect alternative airway equipment readily available to intubate
dynamic airway compression. Images from CT scan are this patient.
static and may not necessarily identify dynamic airway Williams syndrome occurs in 1/20,000 children and
compression, which may occur with the induction of gen- involves a deletion of chromosome 7, which alters the
eral anesthesia. Cardiac echocardiogram can show SVC elastin gene. Patients with Williams syndrome have charac-
or pulmonary outflow tract compression, but flow-volume teristic elfin facies, outgoing personalities, endocrine abnor-
studies are most effective in detecting evidence of dynamic malities, mental retardation, growth deficiency, and altered
airway compression. Depending on the location of this neurodevelopment. Associated cardiovascular pathology

(A) (C)
Exhalation

6.0
Flow (L/sec)

0.0
6.0

(D)
Inhalation

–6.0

Spirometry Prebron- Observed Postbron- Observed Percent


Parameter Units chodilator Percent Pred chodilator Percent Pred Change
FVC Liter 3.53 102 3.63 106 3
FEV1 Liter 2.63 89 2.65 90 0
FEV1/FVCa Percent 74.7 72.8 –3

Pulmonary function test from a child with an intrathoracic airway obstruction (vascular ring). (A)The flow-volume curve shapes suggest
'JHVSFु
a fixed expiratory obstruction. The shape of the inspiratory link is normal; the expiratory flow limb is flattened on both the prebronchodilator
and postbronchodilator flow-volume curves. (B) A magnetic resonance angiogram accompanies the flow loop. (C) Slit-like tracheal compression
before repair. (D) Note the marked improvement in the tracheal lumen after division of the vascular ring. Post, postbronchodilator; Pre,
prebronchodilator; Pred, predicted. (Reprinted with permission from Cote CJ, Lerman J, Todres ID, eds. A Practice of Anesthesia for Infants and
Children. 4th ed. Philadelphia, PA: Saunders Elsevier; 2009:227.)

   r  $ - * / * $ " -  4 $ * & / $ &  0 ' ु " / & 4 5 ) & 4 * "


3 & ' & 3 & /$ &

Slesnick TC, Gertler R, Miller-Hance WC. “Essentials of Cardiology”


in Cote CJ, Lerman J, Todres ID, eds. A Practice of Anesthesia for
Infants and Children. 4th ed. Philadelphia, PA: Saunders Elsevier;
2009:303–4.

31. A N S W E R : A
Many children with congenital heart disease will present for
noncardiac surgeries, so pediatric anesthesiologists should
be well versed in the appropriate hemodynamic goals and
appropriate management of these patients. This scenario
presents an urgent case where a young boy may lose a testis
if his testicular torsion is not corrected in a timely manner.
Tetralogy of Fallot (TOF) is the most common cya-
notic cardiac condition and is characterized by the tet-
rad of pulmonary stenosis, right ventricular outflow tract
obstruction, overriding aorta, and ventricular septal defect
(Fig. 20.3). With TOF, right-to-left shunt results in cya-
Echocardiogram displaying the classic supravalvar aortic
'JHVSFु
nosis, and these patients are prone to “tet spells” or hyper-
narrowing in a patient with Williams syndrome (arrows). Ao V, aortic
cyanotic episodes. Tet spells occur when pulmonary blood
valve; Asc Ao, ascending aorta.
flow is decreased in patients who already have significant
right ventricular outflow tract obstruction. Crying and
includes valvular and supravalvular aortic stenosis, and
coarctation of the aorta. These children also demonstrate
arteriopathy that may involve the origin of the coronary
arteries. Ao
Children with Williams syndrome are predisposed to SVC

increased anesthetic morbidity and potential mortality in


the form of coronary artery stenosis and severe biven- PA
tricular outflow tract obstruction, which may occur with
the induction of general anesthesia. The need for surgery LA
and general anesthesia should be seriously considered, and
thorough cardiac evaluation should be pursued, with special RA
attention to cardiac echocardiogram (Fig. 20.2) and, if pos-
sible, cardiac catheterization data. With potential coronary
LV
artery stenosis in the setting of aortic stenosis with already
compromised coronary artery filling during diastole, this
Pulmonary
patient is at a greatly increased risk of cardiac morbidity and stenosis
mortality with anesthesia.
Right
ventricular
hypertrophy
K E Y FAC T S
r Williams syndrome occurs in 1/20,000 children and RV

involves a deletion of chromosome 7, which alters the


IVC
elastin gene.
r Associated cardiovascular pathology includes valvular Tetralogy of Fallot. This diagram shows the features of the
'JHVSFु
and supravalvular aortic stenosis, and coarctation of tetralogy: VSD, overriding aorta, right ventricular hypertrophy, and
the aorta. pulmonary stenosis. Shown here is both pulmonary and subpulmonary
r Children with Williams syndrome are predisposed to obstruction. The result is right-to-left shunting resulting in cyanosis.
AVC, superior vena cava; RA, right atrium; RV, right ventricle; LA, left
increased anesthetic morbidity and potential mortality
atrium; LV, left ventricle; PA, pulmonary artery; Ao, aorta. (Reprinted
in the form of coronary artery stenosis and severe with permission from: Cote CJ, Lerman J, Todres ID, eds. A Practice of
biventricular outflow tract obstruction, which may Anesthesia for Infants and Children. 4th ed. Philadelphia, PA: Saunders
occur with the induction of general anesthesia. Elsevier; 2009:348.)

    & 7" - 6"5 * 0 /  0 ' ु $ 0 & 9 * 4 5 * / (  % * 4 & " 4 &  r  


inadequate depth of anesthesia are known triggers for tet Patients with testicular torsion should be considered to
spells and exacerbate right-to-left shunting. have full stomachs. To safely anesthetize this TOF patient
Nonpharmacological treatment of tet spells includes with a full stomach, no intravenous access, and an urgent sur-
Valsalva maneuvers and assuming the squatting position. gery requires balancing of anesthetic goals. The most impor-
Further treatment of tet spells includes increasing anes- tant considerations are to increase depth of anesthesia and
thetic depth, systemic vascular resistance, intravascu- inspired oxygen concentration in a timely and safe manner.
lar volume, and inspired oxygen concentration. SVR is Although inhalation induction will likely be slower in
typically increased pharmacologically with phenylephrine. this patient because of reduced pulmonary blood flow
Crystalloid or colloid may be used to increase intravascu- given his TOF physiology, the best way to attenuate this
lar volume. Patients already under general anesthesia may child’s tet spell with the given answer choices is to increase
also benefit from lowered inspiratory ventilator pres- FiO2 and inhaled concentration of sevoflurane. In efforts
sures or beta blockade, as these decrease pulmonary vas- to prevent regurgitation of gastric contents, cricoid pres-
cular resistance and infundibular spasm, respectively. sure should be used during inhalation induction. After the
With regard to the answer choices listed, oral midazolam induction of anesthesia, an intravenous should be quickly
has great bioavailability, but it requires a cooperative patient placed and the patient’s airway secured.
and 10–15 minutes for maximal sedating effects. With supple- A eutectic mixture of local anesthetics (EMLA; 2.5%
mental oxygen, this benzodiazepine will sedate the child so he lidocaine and 2.5% prilocaine) requires 30 minutes to have
will separate from his parents easily and tolerate mask induc- maximal benefit, so this is not the best choice to relieve a
tion, all within the safe time frame to begin this urgent surgery. tet spell in this patient already in the operating room, as
The sympathomimetic effects of intramuscular ket- there will not be enough time to allow this medication to
amine and the intramuscular route of administration are work. If EMLA was not used, or a peripheral intravenous
not ideal in this situation. Although an argument could be was placed prior to maximal effectiveness of EMLA, this
made that intramuscular ketamine will sedate the child, patient would likely continue crying, which would worsen
the intramuscular route of administration will likely cause the infundibular spasm already associated with this
more crying, and the sympathomimetic properties of ket- hypercyanotic episode.
amine may worsen infundibular spasm in this child having Similarly, an intramuscular dose of ketamine would
a hypercyanotic episode. increase the patient’s level of sedation but at the expense of
Further attempts at awake peripheral intravenous place- worsening the hypercyanotic episode, as the intramuscu-
ment are also likely to cause additional pain and crying in lar route of drug delivery would likely worsen crying and
this patient, who is already experiencing a hypercyanotic infundibular spasm because of pain and ketamine’s sympa-
episode. Although increasing intravascular volume is part thomimetic properties.
of the management strategy for this patient, it is more An inhalational induction could theoretically be
important to increase the patient’s level of sedation and expedited via second gas effect from nitrous oxide and
provide increased inspired oxygen content in a timely man- sevoflurane, but decreased FiO2 is not congruent with
ner. It would be irresponsible to not immediately treat this the management strategy for a child having a tet spell, so
patient who requires urgent surgery. Answer C is incorrect.

3 & ' & 3 & /$ & 3 & ' & 3 & /$ &

McEwan A. “Anesthesia for Children Undergoing Heart Surgery” Lönnqvist P, Lerman J. “General Abdominal and Urologic Surgery”
in Cote CJ, Lerman J, Todres ID, eds. A Practice of Anesthesia for in Cote CJ, Lerman J, Todres ID, eds. A Practice of Anesthesia for
Infants and Children. 4th ed. Philadelphia, PA: Saunders Elsevier; Infants and Children. 4th ed. Philadelphia, PA: Saunders Elsevier;
2009:348. 2009:591.

32 . A N S W E R : B 33 . A N S W E R : B
A TOF patient having a hypercyanotic episode, no intra- Although tonsillectomy and adenoidectomy are commonly
venous access, and presenting for urgent surgery demands performed ENT procedures, it would be irresponsible for
thoughtful perioperative consideration. The urgency of this the anesthesiologist caring for this child to not know the
surgery should be reiterated, as the patient’s torsed testis extent of cardiac defect that this child has prior to adminis-
may potentially be saved if its blood supply is restored soon. tering a general anesthetic.
Most testes can be saved if surgery is performed within Trisomy 21 was first described by John Langdon Haydon
6 hours of the onset of pain. This salvage rate decreases to Down in 1887 and is the most common chromosomal
50% if surgery is performed between 6 and 12 hours of the abnormality. It has an incidence of approximately 1/800
onset of pain. live births. Advanced maternal age is a known risk factor

   r  $ - * / * $ " -  4 $ * & / $ &  0 ' ु " / & 4 5 ) & 4 * "


for Down syndrome, and Down syndrome may affect up to and cardiac echocardiogram. This information should be
4% of children born to women over the age of 40. used to tailor this child’s anesthetic, as a Down syndrome
Roughly 40%–50% of patients with Down syndrome patient with complete atrioventricular septal defect may
have some sort of cardiac abnormality. These range from have severe pulmonary hypertension, may have severe
innocent childhood murmurs to complete atrioventricu- conduction abnormalities, and may require inotropic sup-
lar septal defects (AVSDs). Atrioventricular septal defects, port (Fig. 20.4). Elucidation of the extent of this patient’s
or endocardial cushion defects, may be partial or complete. cardiac defect affects anesthetic technique and postopera-
Complete AVSDs typically consist of a large septal defect tive recovery placement.
with atrial and ventricular components and a common atrio- Additional cardiovascular concerns for Down syn-
ventricular valve, and may include conduction abnormalities. drome patients include the propensity for bradycardia
Although a plain chest X-ray may give information with the induction of general anesthesia, and the potential
regarding cardiomegaly, which may reflect congestive heart for pulmonary hypertension, either from cardiac pathol-
failure, this preoperative study does little to confirm the ogy or chronic hypoxia from upper airway obstruction,
patient’s functional or cardiovascular status and is more use- especially in this patient who also has severe obstructive
ful when compared to prior imaging. Moreover, congestive sleep apnea.
heart failure is not suspected in this patient who is growing Other major perioperative concerns for Trisomy 21
well and has no sweating or cyanotic episodes with feeds. include airway and vascular access concerns. Patients with
Similarly, a cardiac stress test is unlikely to yield sub- Trisomy 21 often have chronic pulmonary infections, mac-
stantial clinically important data, as there are no immedi- roglossia with anterior, smaller airways, the potential for
ate concerns regarding myocardial ischemia in this child. atlanto-occipital dislocation, increased incidence of
A cardiac stress test would be more pertinent in a patient postextubation stridor, and potential for difficult vascu-
with coronary artery abnormalities, aortic stenosis, or lar access.
exercise-induced arrhythmias.
Based on the pertinent history obtained by the parents,
this child does not have overt signs of congestive heart fail- K E Y FAC T S
ure, as she feeds well and does not sweat or become cyanotic
with feeds. Children with congestive heart failure may have r Roughly 40%–50% of patients with Down syndrome
nasal flaring, grunting with respirations, chest retrac- have some sort of cardiac abnormality.
tions, poor feeding and growth, and diaphoresis. r Children with congestive heart failure may have nasal
The most appropriate answer selection is to clarify the flaring, grunting with respirations, chest retractions,
child’s cardiac defect and physiology by obtaining an EKG poor feeding and growth, and diaphoresis.

(A) (B)

Ao

PA LA

LA Common A-V
RA value
RA
LV
LV

RV
RV

(A) Diagram of ventricular septal defect showing left-to-right shunt. (B) Diagram of complete atrioventricular septal defect (AVSD)
'JHVSFु
with great vessels removed showing left-to-right shunt through both atrial and ventricular components of the defect and also a single common
atrioventricular valve. RA, right atrium; RV, right ventricle; LA, left atrium; LV, left ventricle; PA, pulmonary artery; Ao, aorta. (Reprinted
with permission from: Cote CJ, Lerman J, Todres ID, eds. A Practice of Anesthesia for Infants and Children. 4th ed. Philadelphia, PA: Saunders
Elsevier; 2009:346.)

    & 7" - 6"5 * 0 /  0 ' ु $ 0 & 9 * 4 5 * / (  % * 4 & " 4 &  r 


r Additional cardiovascular concerns for Down syndrome 3 & ' & 3 & /$ & 4
patients include the propensity for bradycardia with the
induction of general anesthesia and the potential for Ghazal EA, Mason LJ, Coté CJ. “Preoperative Evaluation,
Premedication, and Induction of Anesthesia” in Cote CJ, Lerman J,
pulmonary hypertension, either from cardiac pathology Todres ID, eds. A Practice of Anesthesia for Infants and Children. 4th
or chronic hypoxia from upper airway obstruction. ed. Philadelphia, PA: Saunders Elsevier; 2009:57–8.
Reves JG, Glass PSA, Lubarsky DA, McEvoy MD, Martinez-Ruiz R.
“Intravenous Anesthetics” in Miller RD, Eriksson LI, Fleisher L,
3 & ' & 3 & /$ & Wiener-Kronish JP, eds. Miller’s Anesthesia. 7th ed. Philadelphia,
PA: Churchill Livingstone; 2010:746–7.
Miller-Hance WC. “Anesthesia for Noncardiac Surgery in Children
with Congenital Heart Disease” in Cote CJ, Lerman J, Todres
ID, eds. A Practice of Anesthesia for Infants and Children. 4th ed. 35 . A N S W E R : B
Philadelphia, PA: Saunders Elsevier; 2009:303, 346, 473–4.
Teenagers having surgery are commonly anxious; judicious
use of preoperative anxiolytics may allay this nervousness.
3 4 . A N S W E R : D
Oral or intravenous anxiolytics may be administered in
Patients with autism spectrum disorder have a unique set the preoperative area. Although important to ensure the
of perioperative concerns. Disease states along the autism smooth induction of a general anesthetic, anxiolysis is not
spectrum include milder forms like Asperger’s syndrome the most important concern for this patient and her upcom-
and severe autism, which may be associated with schizo- ing major surgery.
phrenia. When assessing patients with autism spectrum Thoughtful discussions of blood conservation strategies
disorders preoperatively, the patient’s cognitive capacity are the most important part of this patient’s perioperative
and potential effects of psychiatric medications should be discussion and would have ideally begun weeks prior to this
considered. scheduled surgery. Had this discussion begun weeks prior,
Answer A  is not the most reliable method of induc- then the patient could have been started on erythropoi-
ing general anesthesia because intravenous placement is etin and iron supplements to increase her production of
often one of the most challenging aspects of pediatric red blood cells.
anesthesiology, especially for an awake autistic teenager Major blood loss can occur with posterior spinal sur-
who will not even allow caretakers to brush his teeth. This gery because of bleeding from epidural vessels during the
patient likely will not cooperate with an awake intravenous posterior surgical approach. Proper positioning of patients
placement. in the prone position without undue abdominal pressure
Similarly, inhalational induction is not always reli- will avoid venous congestion and may minimize intraop-
able in this patient population. Parental presence or use of erative blood loss attributable to positioning. Other poten-
familiar comfort devices such as headphones may ease an tial strategies to prevent or limit blood loss in a Jehovah’s
autistic patient and make an inhalational induction pos- Witness include the aforementioned preoperative iron and
sible, but this is not the most reliable means of inducing erythropoietin supplementation, appropriate prone posi-
general anesthesia in this patient either. tioning, intraoperative cell savage, use of colloid volume
Although oral midazolam has good bioavailability, it expanders, use of antifibrinolytics epsilon aminocaproic
requires a cooperative patient who will reliably ingest this acid (EACA, Amicar) or transexamic acid, maintenance
potentially bitter-tasting medication, and it requires 10–15 of normothermia, surgical use of fibrin glue, and staged
minutes to work. If this patient does not take the com- surgery or agreeing to stop surgery after a predefined total
plete oral dose of midazolam or spits it out, as may occur blood loss or hematocrit have been determined.
with severely autistic children, then this medication will Some Jehovah’s Witness patients are agreeable to intra-
not work. So Answer C is not the most reliable method of operative cell savage so long as the entire system remains
inducing general anesthesia for this child. in continuity with their vasculature. Whether or not this
Given these answer choices, the most reliable way to patient and her family agree to cell savage is an important
induce general anesthesia with this severely autistic patient is aspect of this patient’s surgical care that should be eluci-
with intramuscular premedication with ketamine, then dated preoperatively. Similarly, the potential use of albumin
inhalational induction with sevoflurane after the patient should be discussed preoperatively. Jehovah’s Witnesses
is sedated. Alternatively, one could also place a peripheral may object to albumin because it is prepared from pools of
intravenous once the patient is adequately sedated, then human plasma, so other plasma expanders like the synthetic
proceed with an intravenous induction. Dosing of intra- colloid hetastarch should be considered. Hetastarch may
muscular ketamine should take into account the antici- be associated with renal insufficiency in select patients, so
pated length of surgery, so as to avoid prolonged hospital its use is controversial.
recovery or time to discharge from overzealous ketamine Antifibrinolytics are lysine analogs that bind plasminogen
administration. and plasmin and effectively prevent fibrinolysis. They have

   r  $ - * / * $ " -  4 $ * & / $ &  0 ' ु " / & 4 5 ) & 4 * "


been shown to significantly decrease intraoperative bleeding with malignant hyperthermia, but idiopathic scoliosis is
in major surgeries like posterior spinal fusion and open heart not (see Box 20.1).
surgery. Transexamic acid has 6 to 10 times the potency of
epsilon aminocaproic acid and has a longer half-life.
Other blood conservation strategies for patients who Box 20.1 463(*$"-"/%"/&45)&5*$
are willing to accept transfusion of blood or blood prod- 13*/$*1-&40'ु#-00%-&44.&%*$*/&
ucts include preoperative autologous donation, acute ."/"(&.&/5
normovolemic hemodilution, and deliberate hypo-
tension. Autologous blood units may be stored for up to
42  days, but this is not a viable option for patients with Preoperative Assessment and Planning
ischemic heart disease or active infections. With acute Management of anemia
normovolemic hemodilution, units of blood are removed Management of anticoagulation and congenital and
at the beginning of surgery and replaced with crystalloid drug-induced coagulopathies
and colloid so that fewer red blood cells are lost per vol- Prophylactic interventional radiology and embolization
ume of blood lost during surgery. These units of blood Prescribing and scheduling of cell salvage apparatus
are then available for transfusion toward the latter por- Restricted diagnostic phlebotomy
tion of surgery. This strategy intentionally decreases
the oxygen-carrying capacity and reduces the margin Intraoperative Blood Conservation
of safety to prevent cerebral and spinal cord ischemia.
Deliberate hypotension is meant to decrease surgical Meticulous surgical hemostasis
bleeding by intentionally decreasing MAP. However, this Blood salvage
technique may risk both cerebral and spinal cord ischemia Hemodilution
and end-organ damage. Moreover, it is unknown exactly Pharmaceutical enhancement of hemostasis
what minimum MAP is required or is safe, as lower MAPs Maintenance of normothermia
also affect neuromonitoring signals. Surgical positioning to minimize blood loss and hypertension
Advances in neuromonitoring have made posterior
spinal surgery safer, but no single neuromonitoring test is Postoperative Blood Conservation
100% reliable. Somatosensory evoked potentials monitor Blood salvage
the integrity of the dorsal columns, and motor evoked Tolerance of anemia
potentials monitor the integrity of the anterior motor por- Optimum fluid and volume management
tion of the spinal cord. Motor evoked potentials have been Restricted diagnostic phlebotomy
associated with complications such as cognitive defects, sei- Adequate analgesia, maintenance of normothermia
zures, bite injuries, intraoperative awareness, scalp burns, Maintain appropriate fluid resuscitation. Significant normo-
and arrhythmias. volemic anemia is well tolerated in hemodynamically stable
Neuromonitoring is affected by anesthetics, so a com- patients.
monly employed balanced anesthetic technique includes In actively bleeding patients, the first management priority
infusions of propofol and fentanyl with <0.5 MAC of vol- must be to stop the bleeding. Avoid attempts to normalize
atile agent for these surgeries. If neuromonitoring signals blood pressure until bleeding is stopped.
are lost intraoperatively, another way to test the integrity Prevent or treat coagulation disorders promptly.
of the patient’s neurologic circuitry is a wake-up test. For a Oral or parenteral iron may be used to improve iron stores.
wake-up test, the patient’s plane of anesthesia is lightened Exogenous erythropoietin therapy effectively increases red
until the patient can follow simple directions and move blood cell mass.
all extremities to command. Once this was demonstrated,
the plane of anesthesia would then be quickly deepened, Hematology and Oncology
and surgery would continue. This test requires a coop-
erative patient and only evaluates gross motor function. Aggressive exogenous erythropoietin therapy and iron ther-
Potential complications of the wake-up test include extu- apy for the prophylaxis of anemia
bation, dislodgment of invasive lines, venous air embo- Individualized chemotherapy protocols to minimize hema-
lism, dislodgment of surgical instrumentation, patient tologic toxicity
injury from falling off the surgical frame, and intraopera- Pharmacologic prophylaxis and treatment of bleeding
tive awareness. Tolerance of anemia
Idiopathic scoliosis does not automatically place this Restricted diagnostic phlebotomy
patient at an increased risk of malignant hyperthermia. Source: From Miller RD, Eriksson LI, Fleisher L, Wiener-Kronish JP,
Central core disease, King-Denborough syndrome, and eds. Miller’s Anesthesia. 7th ed. Philadelphia, PA:  Churchill Livingstone;
Duchenne and other muscular dystrophies are associated 2010:1791.

    & 7" - 6"5 * 0 /  0 ' ु $ 0 & 9 * 4 5 * / (  % * 4 & " 4 &  r  


K E Y FAC T S defects, including interrupted aortic arch, vascular rings,
and hypoplastic left ventricle. The extent and magnitude
r Blood preservation strategies include preoperative iron
of congenital cardiac defects must be elucidated preopera-
and erythropoietin supplementation, appropriate prone
tively with cardiac echocardiogram to tailor the neonate’s
positioning, intraoperative cell savage, use of colloid
appropriate anesthetic technique.
volume expanders, use of antifibrinolytics epsilon
Approximately 60% of DiGeorge patients have charac-
aminocaproic acid (EACA, Amicar) or transexamic acid,
teristic facial abnormalities, including small dysplastic ears,
maintenance of normothermia, surgical use of fibrin
hypertelorism, short downward-slanted palpebral fissures,
glue, and staged surgery or agreeing to stop surgery after
midface hypoplasia, retrognathia, high arched palate, and
a predefined total blood loss or hematocrit has been
bifid uvula. These neonates often present with tetany and
determined.
seizures from hypocalcemia. A detailed electrolyte evalua-
r Other blood conservation strategies include preoperative
tion is helpful to confirm hypocalcemia in this patient who
autologous donation, acute normovolemic hemodilution,
has seizures and tetany, but it is not the most important pre-
and deliberate hypotension.
operative evaluation in this patient.
Large fluid shifts are often associated with major neo-
natal abdominal surgeries like omphalocele repair, but the
3 & ' & 3 & /$ & 4
distractor of potential transfusion of blood products in a
Gertler R, Andropoulos DB. “Cardiopulmonary Bypass and hypocalcemic patient should not lead the reader to incorrectly
Management” in Cote CJ, Lerman J, Todres ID, eds. A Practice choose Answer C instead of Answer B. Similarly, a focused
of Anesthesia for Infants and Children. 4th ed. Philadelphia, airway examination is important, as the majority of patients
PA: Saunders Elsevier; 2009:402–3. with DiGeorge syndrome have dysmorphic facies and may
“Orthopedics:  Scoliosis” in Holzman RS, Mancuso TJ, Sethna NF,
DiNardo JA, eds. Pediatric Anesthesiology Review. New  York, have challenging airways, but the most important preopera-
NY: Springer; 2010:351–5. tive evaluation of this patient is a thorough cardiac evaluation.
Goodnough LT, Monk TG. “Autologous Transfusion, Recombinant Patients with DiGeorge syndrome should be considered
Factor VIIa, and Bloodless Medicine” in Miller RD, Eriksson LI, immunocompromised because thymic hypoplasia dimin-
Fleisher L, Wiener-Kronish JP, eds. Miller’s Anesthesia. 7th ed.
Philadelphia, PA: Churchill Livingstone; 2010:1487–90, 1791. ishes the number and function of their T-cells. Neonates
with DiGeorge syndrome often have recurrent infections
within the first 6 months of life. With this upcoming major
3 6 . A N S W E R : B surgery, blood products that are CMV-negative and irradi-
ated should be available to prevent potential graft versus
The neonate introduced in the question stem has DiGeorge host disease (GVHD) from blood product transfusion. In
syndrome. If the clinical diagnosis of DiGeorge syndrome addition, strict aseptic technique should be followed during
was not immediately made, the fact that this neonate vascular access.
requires omphalocele repair should prompt a thorough
cardiac evaluation.
Omphalocele occurs in 1/6000 births and is com- K E Y FAC T S
monly associated with congenital heart disease, bladder r Omphalocele occurs in 1/6000 births and is commonly
extrophy, and Beckwith-Widemann syndrome. The pri- associated with congenital heart disease, bladder
mary defect with omphalocele is a midline failure of gut extrophy, and Beckwith-Widemann syndrome.
migration from the yolk sac into the abdomen. Large fluid r DiGeorge syndrome occurs in approximately
shifts often occur with omphalocele repairs, and although 1/3000–5000 live births and is the second most
ensuring appropriate volume status and urine output are common genetic cause of congenital heart defects.
important, this is not as important as a thorough preopera- r Patients with DiGeorge syndrome should be considered
tive cardiac evaluation for this patient with both DiGeorge immunocompromised because thymic hypoplasia
syndrome and omphalocele. diminishes the number and function of their T-cells.
DiGeorge syndrome, conotruncal face, and velocardio-
facial syndromes are encompassed by 22q11.2 deletion syn-
drome, which is also known as CATCH 22 syndrome. The 3 & ' & 3 & /$ & 4
acronym CATCH 22 includes cardiac defects, abnormal
facies, thymic hypoplasia, cleft palate, and hypocalcemia, “DiGeorge Syndrome” in Bissonnette B, Luginbuehl I, Marciniak
and involves deletions of chromosome 22. B, Dalens B, eds. Syndromes:  Rapid Recognition and Perioperative
DiGeorge syndrome occurs in approximately Implications. Online ed. New York, NY: McGraw-Hill; 2006.
Roberts Jr JD, Romanelli TM, Todres ID. “Neonatal Emergencies”
1/3000–5000 live births and is the second most common in Cote CJ, Lerman J, Todres ID, eds. A Practice of Anesthesia for
genetic cause of congenital heart defects. Patients with Infants and Children. 4th ed. Philadelphia, PA: Saunders Elsevier;
DiGeorge syndrome have a variety of congenital heart 2009:304, 765.

   r  $ - * / * $ " -  4 $ * & / $ &  0 ' ु " / & 4 5 ) & 4 * "


37. A N S W E R : B with repeated bladder catheterization. Strict adherence to
latex-free environments must be followed.
Most children with myelomeningocele present for closure
within the first 48 hours of life to minimize risk of infec- K E Y FAC T S
tion. As with this neonate, many are scheduled for surgical
repair prenatally because the defect is usually detected on r Major anesthetic considerations for repair of
ultrasound. Major anesthetic considerations for this sur- myelomeningoceles include positioning, potential
gery include positioning, potential for massive fluid and for massive fluid and blood loss, and postoperative
blood loss, and postoperative respiratory difficulties. respiratory difficulties.
With such a large posterior defect, it is important to r For very large defects, it may be safest to induce general
ensure that there is no direct pressure on the myelome- anesthesia and intubate with the patient in the lateral
ningocele during the induction of general anesthesia and decubitus position, so as to avoid undue pressure on the
while securing the neonatal airway. Depending on the size lesion.
of the defect, general anesthesia may be induced with the r Hydrocephalus, tethered cord, and type II Arnold-
area surrounding the patient’s defect well padded in the Chiari malformations often accompany lesions like
supine position, or alternately induction and intubation myelomeningocele, and ventilatory responses to
may occur in the lateral decubitus position. For very large hypoxia and hypercarbia may be diminished or absent
defects, it may be safest to induce general anesthesia and postoperatively.
intubate with the patient in the lateral decubitus position,
so as to avoid undue pressure on the lesion. 3 & ' & 3 & /$ & 4
With myelomeningocele repair, there is the potential
for considerable blood and fluid loss. Because a large por- McClain CD, Soriano SG, Rockoff MA. “Pediatric Neurosurgical
tion of the baby’s skin surface area will be exposed for sur- Anesthesia” in Cote CJ, Lerman J, Todres ID, eds. A Practice
gery, maintaining adequate volume status and euthermia of Anesthesia for Infants and Children. 4th ed. Philadelphia,
PA: Saunders Elsevier; 2009:530–2.
are important perioperative considerations, and third space “Neuroanesthesia” in Holzman RS, Mancuso TJ, Sethna NF, DiNardo
losses can be considerable. JA, eds. Pediatric Anesthesiology Review. New York, NY: Springer;
All of the answer choices are valid concerns for a neo- 2010:182–3.
nate undergoing myelomeningocele repair and VP shunt Coté CJ. “Pediatric Anesthesia” in Miller RD, Eriksson LI, Fleisher L,
Wiener-Kronish JP, eds. Miller’s Anesthesia. 7th ed. Philadelphia,
placement, but the most important reason for recovering PA: Churchill Livingstone; 2010:2589.
this patient in the ICU is to monitor his respiratory status
postoperatively.
3 8 . A N S W E R : A
Patients with myelomeningocele often have
sleep-disordered breathing, central apnea, hypoventila- This question highlights why a single-shot spinal is consid-
tion, and obstructive sleep apnea. Since this baby is <60 ered a preferred anesthetic technique for a former preterm
weeks postconceptual age, he is already at an increased risk of infant. By using a complete regional technique, one can
postoperative apnea. Coupled with potential postoperative potentially avoid administering narcotics and a general
respiratory challenges following myelomeningocele repair anesthetic, both of which may contribute to postoperative
and VP shunt placement, it would be advisable to monitor apnea in this former premature twin whose postconcep-
this patient in an ICU level setting postoperatively, since he tual age is <60 weeks. Unfortunately, it appears that his
now has multiple reasons to have sleep-disordered breathing. single-shot spinal has worn off.
Hydrocephalus, tethered cord, and type II Answer A  would prevent postoperative apnea in this
Arnold-Chiari malformations often accompany lesions patient. It should be clearly communicated to the surgeon
like myelomeningocele. Moreover, if this neonate also has a what dose of local anesthetic is safe and appropriate for this
type II Arnold-Chiari malformation that contributes to his 4  kg infant, as a small volume of local anesthetic quickly
hydrocephalus and need for a VP shunt, then his ventila- becomes a toxic dose for small infants receiving bupiva-
tory response to hypoxia and hypercarbia may be dimin- caine. Since the infant has already received 0.4 cc of 0.5%
ished or absent postoperatively. In addition, depending on bupivacaine in the subarachnoid space, the surgeon should
the degree of hydrocephalus, this neonate may have the be limited to 3.2 cc of 0.25% bupivacaine for infiltration.
potential for brainstem herniation and may also have a If no narcotics are administered, the patient will be at no
cranial nerve palsy that would result in inspiratory stridor. increased risk of postoperative apnea with this strategy.
This patient thus has several reasons to have his respiratory Answer B does not prevent postoperative apnea. It
status closely monitored in the ICU setting postoperatively. makes no difference what narcotic is administered. Any
Patients with myelodysplasias like myelomeningocele administration of narcotic has been associated with
are at an increased risk of developing latex sensitiv- postoperative apnea in former preterm infants with a
ity because of repeated exposures to latex, such as occurs postconceptual age <60 weeks.

    & 7" - 6"5 * 0 /  0 ' ु $ 0 & 9 * 4 5 * / (  % * 4 & " 4 &  r   


Answer C does not reliably prevent postoperative apnea and Children. 4th ed. Philadelphia, PA:  Saunders Elsevier;
either. Even if no narcotic medication is administered, for- 2009:877–9.
Welborn LG, Rice LJ, Hannallah RS, Broadman LM, Ruttimann UE,
mer preterm infants with a postconceptual age <60 weeks Fink R. Postoperative apnea in former preterm infants:  prospec-
undergoing general anesthesia are at increased risk of tive comparison of spinal and general anesthesia. Anesthesiology.
postoperative apnea. 1990;72:838–42.
Answer D is attractive since the N-methyl-D-aspartate
(NMDA) antagonist ketamine is often used to maintain 39. A N S W E R : B
spontaneous respirations while also providing analgesia.
However, when ketamine is used to supplement neonatal Pyloric stenosis is a medical emergency, not a surgical emer-
spinal anesthesia, the incidence of postoperative apnea is gency. It is common in firstborn males within the first 3
actually greater than with a general anesthetic. to 6 weeks of life. Perioperative concerns for this patient
Spinal anesthesia is an attractive option for this patient include full stomach precautions and severe dehydration.
and surgery because of its association with less postopera- Volume status for patients with pyloric stenosis coming to
tive apnea, and timing of surgery meshes well with unilat- the operating room must be adequate, and metabolic alka-
eral inguinal hernia repair. There is a lower risk of total losis from repeated vomiting should be corrected with
spinal in neonates, but there are several technical consid- vigorous fluid resuscitation prior to surgical correction.
erations when performing a neonatal spinal. Among these Answer B best indicates that the patient is ready for
considerations is the appropriate positioning and bracing surgery, as his electrolyte imbalances have been corrected.
of the baby while the spinal is being performed. Excessive Specifically, his hypokalemia, hypochloremia, and
neck flexion should be avoided, as this may obstruct increased bicarbonate level from his contraction alkalo-
the patient’s airway. Since the conus medullaris is located sis have been appropriately corrected with fluid resuscita-
at L3 in neonates and infants, spinal anesthesia should be tion. Other signs of adequate volume status in this patient
performed at the L4/5 or L5/S1 level. When the subarach- include appropriate urine output (wet diapers), normal
noid space has been entered, the local anesthetic should be behavior (not irritable or lethargic), normal skin turgor,
administered slowly with a tuberculin syringe. tears if the child is crying, and anterior fontanelles that are
Any repositioning of the infant after placement of the not depressed.
spinal should be performed carefully. In particular, any Answer A is incorrect because the patient’s potassium,
Trendelenburg position should be avoided, as sometimes chloride, and bicarbonate levels have not yet been cor-
occurs when a neonate’s legs are lifted to place an electro- rected. These laboratory values are consistent with meta-
cautery pad on the patient’s back. This seemingly innocu- bolic alkalosis and likely are the values that this infant
ous maneuver could potentially cause cephalad spread of with pyloric stenosis initially presented with. Neither
the local anesthetic and a resultant high spinal. Answer C nor D is correct because the bicarbonate levels
have not been appropriately corrected with adequate fluid
resuscitation.
K E Y FAC T S Patients with pyloric stenosis have a full stomach that
r By using a complete regional technique, one can may contain radiocontrast dye from confirmatory imag-
potentially avoid administering narcotics and a ing studies. It is advisable to suction the stomach with a
general anesthetic, both of which may contribute to large-bore catheter with the patient in the supine, right,
postoperative apnea in this former premature twin and left lateral decubitus positions. One study found that
whose postconceptual age is <60 weeks. this technique removes 98% of gastric contents. After
r Any administration of narcotic has been associated with the stomach has been suctioned, options for intubation
postoperative apnea in former preterm infants with a include awake or rapid sequence intubation. Either
postconceptual age <60 weeks. technique is reasonable, but a study comparing awake
r Former preterm infants with a postconceptual age <60 versus rapid sequence intubation demonstrated increased
weeks undergoing general anesthesia are at increased risk success with first pass intubation when muscle relaxant
of postoperative apnea. was used.
r When ketamine is used to supplement neonatal spinal Postoperative analgesia for laparoscopic or open pyloro-
anesthesia, the incidence of postoperative apnea is myotomy typically consists of local anesthetic infiltration
actually greater than with a general anesthetic. and acetaminophen. Narcotics are not routinely required
for postoperative analgesia for this surgery.

3 & ' & 3 & /$ & 4 K E Y FAC T S


Everett LL, Fuzaylov G, Todres ID. “Pediatric Emergencies” in Cote r Perioperative concerns for patients with pyloric stenosis
CJ, Lerman J, Todres ID, eds. A Practice of Anesthesia for Infants include full stomach precautions and severe dehydration.

   r  $ - * / * $ " -  4 $ * & / $ &  0 ' ु " / & 4 5 ) & 4 * "


r Metabolic alkalosis (hypokalemia, hypochloremia, and incidence of laryngospasm in patients with active URIs is
increased bicarbonate) from repeated vomiting should similar independent of endotracheal tube versus LMA use.
be corrected with vigorous fluid resuscitation prior to LMA use may decrease this child’s propensity for broncho-
surgical correction. spasm, but it will not decrease the chance of laryngospasm
r Signs of adequate volume status in this patient include in this patient with an active URI.
appropriate urine output (wet diapers), normal behavior Answer B is incorrect. A  study found that healthy
(not irritable or lethargic), normal skin turgor, tears if children with URIs undergoing surgery not involving the
the child is crying, and anterior fontanelles that are not airway or body cavity did not benefit from preoperative
depressed. bronchodilator administration of either albuterol or
ipratropium.
Answer C is incorrect. There are no data to support the
3 & ' & 3 & /$ & 4 routine use of steroids and broad-spectrum antibiotics in
children with active URIs.
Cook-Sather SD, Liacouras CA, Previte JP, Markakis DA, Schreiner Unless this surgery is urgent or emergent, it should be
MS. Gastric fluid measurement by blind aspiration in pae-
diatric patients:  a gastroscopic evaluation. Can J Anaesth.
postponed. Active URIs in children scheduled for cardiac
1997;44:168–72. surgery warrant special consideration, as there are risks that
Roberts Jr JD, Romanelli TM, Todres ID. “Neonatal Emergencies” the child’s heart or clinical disease process will deteriorate
in Cote CJ, Lerman J, Todres ID, eds. A Practice of Anesthesia for while awaiting the child to recover from an active URI. The
Infants and Children. 4th ed. Philadelphia, PA: Saunders Elsevier;
2009:761–2.
child scheduled for cardiac surgery should be optimized
Coté CJ. “Pediatric Anesthesia” in Miller RD, Eriksson LI, Fleisher L, preoperatively, and a thorough discussion of the periopera-
Wiener-Kronish JP, eds. Miller’s Anesthesia. 7th ed. Philadelphia, tive risks of an active URI with general anesthesia should be
PA: Churchill Livingstone; 2010:2591. explained to the child’s parents.
An active URI was determined to be an independent
4 0 . A N S W E R : D risk factor for postoperative infections and other post-
operative complications in children undergoing cardiac
Pediatric anesthesiologists must be able to anticipate and surgery. In a prospective study of children scheduled for
recognize signs and symptoms of active upper respiratory cardiac surgery, overall incidence of respiratory adverse
infections. This former preterm child has several symptoms events, multiple postoperative complications, and bacte-
of an active URI. Since this surgery is elective, it should be rial infection were significantly higher in children with
postponed at least 4 weeks until symptoms of this active active URIs. Postoperative ICU duration was higher
URI have resolved. Moreover, since this child has not in the URI group, but duration of hospital stay was not
received any vaccinations, he may also have a more serious significantly increased among children with or without
illness like varicella or measles, and this should be investi- active URIs.
gated further. There are rare instances when postponing surgery is
Other predictors for an acute URI include parental not an option, such as occurs with solid organ transplant,
reports of the child having a “cold,” nasal congestion, snor- and the risks should be explained and documented, and
ing, exposure to secondhand smoking, and parental reports vigilance should be employed to watch for potential
that the child appears ill, which indicate changes in mental respiratory complications. Recognizing URI signs and
status or behavior. symptoms and appropriately balancing risks and ben-
Having an active URI increases this child’s risks of efits for each patient are fundamental in the clinical
perioperative airway complications such as broncho- acumen of pediatric anesthesiologists (see Box 20.2 and
spasm, laryngospasm, and major desaturation episodes Table 20.5).
up to 10-fold. Furthermore, since he is a former preterm
infant, his risks of respiratory complications are increased.
These risks should be highlighted and discussed with the 3 & ' & 3 & /$ & 4
child’s parents in the preoperative area, as cancellation of
surgery may impose emotional and economic burdens on Ghazal EA, Mason LJ, Coté CJ. “Preoperative Evaluation,
the parents with the challenges of rescheduling elective Premedication, and Induction of Anesthesia” in Cote CJ, Lerman J,
Todres ID, eds. A Practice of Anesthesia for Infants and Children. 4th
surgery. ed. Philadelphia, PA: Saunders Elsevier; 2009:60–2.
Answer A is incorrect because this patient has an active Malviya S, Voepel-Lewis T, Siewert M, Pandit UA, Riegger LQ, Tait
URI and has other risk factors that increase his likelihood AR. Risk factors for adverse postoperative outcomes in children pre-
of having perioperative respiratory complications, even if a senting for cardiac surgery with upper respiratory tract infections.
Anesthesiology. 2003;98:628–32.
laryngeal mask airway is used instead of tracheal intubation. Rolf N, Coté CJ. Frequency and severity of desaturation events during
Bronchospasm has been found to occur more frequently general anesthesia in children with and without upper respiratory
in patients with active URIs who are intubated. Overall infections. J Clin Anesth. 1992;4:200–3.

    & 7" - 6"5 * 0 /  0 ' ु $ 0 & 9 * 4 5 * / (  % * 4 & " 4 &  r   


Box 20.2 %*''&3 &/5*"-%*"(/04*40'ु" peripheral perfusion. The accumulation of sickled eryth-
$)*-%8*5)ु"36//:ु/04& rocytes causes ischemia, further sickling, and eventually
end-organ failure.
Noninfectious Causes Many organ systems are affected by sickle cell disease,
and patients with this disease are predisposed to acute chest
Allergic rhinitis:  seasonal, perennial, clear nasal discharge,
syndrome (ACS), recurrent episodes of vaso-occlusive cri-
no fever
ses, infection, renal insufficiency, osteonecrosis, and chole-
Vasomotor rhinitis: emotional (crying), temperature
lithiasis. Chronic pulmonary and neurologic diseases like
changes
stroke are causes of significant morbidity and mortality in
this patient population.
Infectious Causes
In the perioperative period, the most common compli-
Viral infections cations in children with SCD include ACS, fever, infec-
Nasopharyngitis (common cold) tion, vaso-occlusive crisis, and transfusion-related
Flu syndrome (upper and lower respiratory tract) events. Because this patient is predisposed to ACS and
Laryngotracheal bronchitis (infectious croup) vaso-occlusive crisis, liberal analgesics should be adminis-
Viral exanthems tered, and regional techniques should be considered.
Measles Similarly, hypoxia should be avoided by ensuring
Chickenpox adequate oxygenation and oxygen-carrying capacity of
Acute bacterial infections hemoglobin, usually on the order of 10 g/dL. Transfusion
Acute epiglottitis of blood corrects anemia, dilutes the concentration of HbS
Meningitis erythrocytes, and can prevent severe complications like
Streptococcal tonsillitis stroke. Since sickle cell patients often receive multiple trans-
fusions of blood products, it is important that the blood
they receive is phenotype matched, leukocyte reduced,
Table 20.5 '"$5034"''&$5*/(%&$*4*0/
and screened for sickle cells, as these patients have a high
'03ु&-&$5*7&463(&3:*/ु$)*-%8*5)ु611&3
rate and risk of alloimmunization.
3 &41*3 "503:53 "$5*/'&$5*0/
Aside from hypoxia, acidosis and circulatory stasis also
contribute to sickling in susceptible individuals. Acid-base
PRO CON status should be closely monitored, and circulatory status
$IJMEIBTiKVTUBSVOOZOPTF uOP 1BSFOUTDPOđSN can be optimized by providing appropriate fluid resuscita-
PUIFSTZNQUPNT iNVDICFUUFSu TZNQUPNTुGFWFS NBMBJTF  tion and avoiding excessive or prolonged tourniquet use.
DPVHI QPPSBQQFUJUF KVTU Because patients with SCD are already at an increased
EFWFMPQFETZNQUPNTMBTU risk of developing stroke, hyperventilating to an end-tidal
OJHIU
CO2 < 25  mm Hg is not advised, as that would further
"DUJWFBOEIBQQZDIJME -FUIBSHJD JMMBQQFBSJOH
compromise already tenuous cerebral blood flow.
$MFBSSIJOPSSIFB 1VSVMFOUOBTBMEJTDIBSHF Older patients with SCD like this patient may develop
$MFBSMVOHT 8IFF[JOH SBMFT restrictive lung disease and pulmonary hypertension.
0MEFSDIJME $IJMEुZFBS FYQSFFNJF This results from repeated ACS-like injury, chronic inflam-
4PDJBMJTTVFTुIBSETIJQGPSQBSFOUT 0UIFSGBDUPSTुIJTUPSZPG mation, and deficiency of nitric oxide. In addition, chronic
UPCFBXBZGSPNXPSL JOTVSBODF SFBDUJWFBJSXBZEJTFBTF NBKPS intravascular hemolysis decreases the production of NO,
XJMMSVOPVU PQFSBUJPO FOEPUSBDIFBMUVCF and increased scavenging of NO reduces its bioavailability.
SFRVJSFE Endothelial dysfunction and NO deficiency cause com-
SOURCE: Reprinted with permission from: Cote CJ, Lerman J, Todres ID, plications like pulmonary hypertension, priapism, and skin
eds. A Practice of Anesthesia for Infants and Children. 4th ed. Philadelphia,
PA: Saunders; 2009: 61.
ulceration. Studies have shown that inhaled NO amelio-
rates the symptoms of ACS, so inhaled nitric oxide would
not be deleterious for this patient and may potentially
41. A N S W E R : C improve ventilation (see Box 20.3).
Sickle cell disease (SCD) encompasses a group of inher-
ited hemoglobinopathies, with HbSC (sickle cell/HbC)
and HbSD (sickle cell/HbD) having the greatest potential 3 & ' & 3 & /$ & 4
for significant clinical sickling. This sickling occurs when
Haberkern CM, Webel NE, Eisses MJ, Bender MA. “Essentials of
sickled erythrocytes accumulate in the microvascular of Hematology” in Cote CJ, Lerman J, Todres ID, eds. A Practice
susceptible patients in the setting of factors that promote of Anesthesia for Infants and Children. 4th ed. Philadelphia,
the deoxygenation of hemoglobin and interferes with PA: Saunders Elsevier; 2009:182–6.

  r $-* / *$ " -4$* & /$&0'ु" / & 45 ) & 4* "


Box 20.3 1&3*01&3 "5*7&$0/4*%&3 "5*0/4 Fischer SP, Bader AM, Sweitzer B. “Preoperative Evaluation” in Miller
RD, Eriksson LI, Fleisher L, Wiener-Kronish JP, eds. Miller’s
"/%$0/$&3 /4*/ु4*$,-&$&--%*4&"4& Anesthesia. 7th ed. Philadelphia, PA:  Churchill Livingstone;
2010:1028.
Preoperative
Screening if unknown status in at-risk children 4 2 . A N S W E R : C
Admission to hematology service (under most circumstances)
History of acute chest syndrome, vaso-occlusive pain crises, Beckwith-Wiedemann syndrome usually occurs spo-
hospitalizations, transfusions, transfusion reactions radically and consists of exomphalos, macroglossia,
Neurologic assessment (strokes, cognitive limitations) gigantism, organomegaly, and hypoglycemia. It has
History of analgesic and other medication use an incidence of approximately 1.5 per 100,000 with an
Hematocrit increased incidence in West India, and occurs as a result
Oxygen saturation (room air), chest radiograph of failure of normal biparental inheritance of chromo-
Pulmonary function tests (when appropriate) some 11p15, which contains growth-promoting and
Echocardiography (when appropriate) tumor-suppressor genes, resulting in increased insulin-like
Neurologic imaging (when there are recent changes) growth factor 2 (IGF-2). This increased IGF-2 is responsi-
Renal function studies ble for somatic overgrowth, the predisposition for tumors,
Transfusion crossmatch (antibody-matched, leukocyte and hypoglycemia.
reduced, sickle negative) As with this patient, early visceromegaly occurs, and
Transfusion to correct anemia (under most circumstances) neonates with Beckwith-Wiedemann syndrome often pres-
Parenteral hydration while NPO ent for repair of omphalocele, anomalies of intestinal rota-
Pain management tion, and diaphragmatic eventration. Sometimes patients
Aggressive bronchodilator therapy with Beckwith-Wiedemann syndrome have midface hypo-
Appropriate antibiotic therapy, including pre-splenectomy plasia, so a full preoperative airway evaluation is impor-
antibiotics and immunizations, where indicated tant, as securing this patient’s airway may be challenging. It
would be prudent to have a difficult airway cart and vari-
Intraoperative ous LMAs available for instrumenting this child’s airway.
Several organ systems may be involved with
Maintenance of oxygenation, perfusion, normal acid-basis Beckwith-Wiedemann syndrome. Cardiomegaly, hepa-
status, temperature, hydration tosplenomegaly, visceromegaly, and several malignant
Availability of appropriately prepared blood (when tumors have been associated with Beckwith-Wiedemann
appropriate) syndrome. When patients with Beckwith-Wiedemann
Replacement of blood loss syndrome have hemihypertrophy, an increased incidence
Anesthetic technique appropriate for procedure and postop- of malignant tumors like adrenal carcinoma, Wilms
erative analgesic requirements tumor, hepatoblastoma, thoracic neuroblastoma, rhab-
Physiologic effects of laparoscopy on circulatory and respira- domyosarcoma, and congenital gastric teratoma has been
tory function reported. Proper preoperative workup of this patient with
Appropriate antibiotic therapy Beckwith-Wiedemann syndrome and hemihypertrophy
Judicious use of tourniquets, cell saver, and cardiopulmonary thus includes screens for somatic tumors and a full car-
bypass diac workup.
Answer C is correct, as a cardiac echocardiogram will
Postoperative evaluate for cardiomegaly, cardiomyopathy, congenital
Management by hematology service cardiac lesions, or cardiac hamartoma. Data from this
Monitoring for complications, especially acute chest syn- study will more strongly influence decisions regarding this
drome and vaso-occlusive pain crises patient’s anesthetic and choice of intraoperative medica-
Maintenance of oxygen saturation monitoring and supple- tions. Inotropes and bacterial endocarditis prophylaxis may
mentation as needed, including supplemental oxygen the be indicated for this patient, depending on what her cardiac
first 24 hours regardless of oxygen saturation echocardiogram shows.
Appropriate hydration (oral + parenteral) Answer A  is incorrect. Although it would be nice to
Appropriate antibiotic therapy know the patient’s fasting glucose level, this data will not
Aggressive pain management significantly alter the patient’s anesthetic, as this data
Early mobilization point is not absolutely required to maintain intraoperative
euglycemia.
Similarly, Answer B is incorrect. Beckwith-Wiedemann
syndrome is associated with hepatosplenomegaly, but

    & 7" - 6"5 * 0 /  0 ' ु $ 0 & 9 * 4 5 * / (  % * 4 & " 4 &  r  


knowing the patient’s LFTs will not significantly alter her r Proper preoperative workup of this patient with
anesthetic plan. If a neuraxial block were planned for post- Beckwith-Wiedemann syndrome and hemihypertrophy
operative analgesia, then coagulation studies and a platelet thus includes screens for somatic tumors and a full
count could alter the anesthetic plan. However, this data cardiac workup.
will not change the decision regarding the planned femoral
peripheral nerve block catheter.
Answer D is a distractor, as Beckwith-Wiedemann syn- 3 & ' & 3 & /$ & 4
drome has not been associated with susceptibility to malig-
nant hyperthermia. “Beckwith-Wiedemann Syndrome” in Bissonnette B, Luginbuehl I,
Marciniak B, Dalens BJ. eds.Syndromes:  Rapid Recognition and
Perioperative Implications. New  York, NY:  McGraw-Hill; 2006.
K E Y FAC T S http://accessanesthesiology.mhmedical.com/content.aspx?bookid=
852&Sectionid=49517273. Accessed July 11, 2014.
r Beckwith-Wiedemann syndrome usually occurs Wheeler M, Coté CJ, Todres ID. “The Pediatric Airway.” Cote CJ,
Lerman J, Todres ID. A Practice of Anesthesia for Infants and
sporadically and consists of exomphalos, macroglossia, Children. 4th ed. Philadelphia, PA: Saunders Elsevier; 2009:274.
gigantism, organomegaly, and hypoglycemia.
r Beckwith-Wiedemann syndrome is associated with
midface hypoplasia, and airway management can be
difficult.

    r $ - * / * $ " -  4 $ * & / $ &  0 ' ु " / & 4 5 ) & 4 * "

You might also like